Appearance
❓:In the Cartesian coordinate system xOy, let AB be a chord of the parabola y^2 = 4x passing through the point F(1,0). The circumcircle of triangle AOB intersects the parabola at point P (different from points O, A, B). If PF bisects angle APB, find all possible values of |PF|.
🧠:Okay, let's see. I need to solve this geometry problem involving a parabola, a chord through a specific point, a circumcircle, and angle bisectors. Hmm. Let's break it down step by step.First, the problem states: In the Cartesian coordinate system xOy, AB is a chord of the parabola y² = 4x passing through the point F(1,0). The circumcircle of triangle AOB intersects the parabola again at point P (different from O, A, B). If PF bisects angle APB, find all possible values of |PF|.Alright, so we have the parabola y² = 4x. The vertex is at the origin O(0,0), and the focus is at (1,0), which is point F. So chord AB passes through the focus F. Then, the circumcircle of triangle AOB meets the parabola again at P, and PF is the angle bisector of angle APB. We need to find |PF|.Let me start by recalling some properties of parabolas and circumcircles. For the parabola y² = 4x, any chord passing through the focus has some specific properties. Maybe the latus rectum? Wait, the latus rectum is the chord through the focus perpendicular to the axis of the parabola, which in this case is the x-axis. So the latus rectum is y² = 4x at x=1, so y²=4*1=4, so y=±2. So the latus rectum is from (1,2) to (1,-2). But AB is any chord through F(1,0), not necessarily the latus rectum.So AB is a general chord through F. Let me parametrize points A and B on the parabola. Since it's a parabola y²=4x, we can represent points on it in parametric form. Let me recall that parametric equations for y²=4x are x = at², y = 2at, where a=1 in this case. So parametric coordinates can be (t², 2t). So point A can be (t₁², 2t₁) and point B can be (t₂², 2t₂). Since chord AB passes through F(1,0), we can use the equation of chord AB to find a relationship between t₁ and t₂.Equation of chord AB joining points (t₁², 2t₁) and (t₂², 2t₂). The slope of AB is (2t₂ - 2t₁)/(t₂² - t₁²) = 2(t₂ - t₁)/[(t₂ - t₁)(t₂ + t₁)] = 2/(t₁ + t₂). So slope is 2/(t₁ + t₂). The equation of the line can be written using point-slope form. Let's pick point A: y - 2t₁ = [2/(t₁ + t₂)](x - t₁²). Since chord passes through F(1,0), substitute x=1, y=0 into the equation:0 - 2t₁ = [2/(t₁ + t₂)](1 - t₁²)=> -2t₁ = [2(1 - t₁²)]/(t₁ + t₂)Multiply both sides by (t₁ + t₂):-2t₁(t₁ + t₂) = 2(1 - t₁²)Divide both sides by 2:- t₁(t₁ + t₂) = 1 - t₁²Expand left side:- t₁² - t₁ t₂ = 1 - t₁²Add t₁² to both sides:- t₁ t₂ = 1Thus, t₁ t₂ = -1.So the product of parameters t₁ and t₂ is -1. That's a key relation.So points A(t₁², 2t₁) and B(t₂², 2t₂) with t₁ t₂ = -1. So t₂ = -1/t₁. Let's write t₂ as -1/t₁. Then point B is ((-1/t₁)², 2*(-1/t₁)) = (1/t₁², -2/t₁).So AB is determined by parameter t₁. Now, next step: circumcircle of triangle AOB intersects the parabola again at P ≠ O, A, B. So we need to find point P on the parabola and on the circumcircle of AOB.Then, given that PF bisects angle APB, find |PF|.So first, maybe find the equation of the circumcircle of triangle AOB, then find its intersection with the parabola (other than O, A, B), which is point P. Then, use the angle bisector condition to get equations for coordinates of P, leading to |PF|.But since this seems complex, perhaps there's a smarter way using properties of parabolas and circles, or some symmetry.Alternatively, parametrize everything in terms of t₁ and proceed.Let me try the parametric approach.Let me denote t₁ as t for simplicity. Then t₂ = -1/t.So points:O(0,0),A(t², 2t),B(1/t², -2/t).Now, find the circumcircle of triangle AOB. The circumcircle passes through O, A, B. Let's find its equation.General equation of a circle: x² + y² + Dx + Ey + F = 0.Since it passes through O(0,0):0 + 0 + 0 + 0 + F = 0 ⇒ F = 0.So the circle equation is x² + y² + Dx + Ey = 0.Now, plug in point A(t², 2t):(t²)² + (2t)^2 + D(t²) + E(2t) = 0t^4 + 4t² + D t² + 2E t = 0Similarly, plug in point B(1/t², -2/t):(1/t²)^2 + (-2/t)^2 + D(1/t²) + E(-2/t) = 01/t^4 + 4/t² + D/t² - 2E/t = 0Multiply both sides by t^4 to eliminate denominators:1 + 4t² + D t² - 2E t³ = 0So we have two equations:From point A: t^4 + 4t² + D t² + 2E t = 0From point B: 1 + 4t² + D t² - 2E t³ = 0Let me write them as:1) t^4 + (4 + D) t² + 2E t = 02) (4 + D) t² - 2E t³ + 1 = 0Hmm, seems a bit complicated. Maybe solve for D and E?Let me denote equation 1 and equation 2.From equation 1: t^4 + (4 + D) t² + 2E t = 0From equation 2: (4 + D) t² - 2E t³ + 1 = 0Let me solve equation 1 for (4 + D) t²:(4 + D) t² = -t^4 - 2E tPlug this into equation 2:(-t^4 - 2E t) - 2E t³ + 1 = 0=> -t^4 - 2E t - 2E t³ + 1 = 0Let me rearrange:- t^4 - 2E t³ - 2E t + 1 = 0Hmm. Not sure. Maybe express E from equation 1?From equation 1: 2E t = - t^4 - (4 + D) t²But that still involves D. Alternatively, maybe find a relation between equations 1 and 2.Alternatively, let's consider that equations 1 and 2 must hold for the same t. So we can set up a system of equations to solve for D and E.Let me write equations 1 and 2 as:1) t^4 + (4 + D) t² + 2E t = 02) (4 + D) t² - 2E t³ + 1 = 0Let me denote S = (4 + D) t². Then equation 1 becomes t^4 + S + 2E t = 0, so S = -t^4 - 2E t.Equation 2 is S - 2E t³ + 1 = 0.Substitute S from equation 1 into equation 2:(-t^4 - 2E t) - 2E t³ + 1 = 0=> -t^4 - 2E t - 2E t³ + 1 = 0Factor terms with E:- t^4 - 2E(t³ + t) + 1 = 0Express this as:2E(t³ + t) = - t^4 + 1Thus,E = (- t^4 + 1)/(2(t³ + t)) = (1 - t^4)/(2 t (t² + 1))Simplify numerator: 1 - t^4 = (1 - t²)(1 + t²) = (1 - t)(1 + t)(1 + t²)Denominator: 2 t (t² + 1)So E = [ (1 - t)(1 + t)(1 + t²) ] / [ 2 t (t² + 1) ) ] = (1 - t²)(1 + t²)/(2 t (t² + 1)) ) = (1 - t²)/(2 t )Wait, because (1 + t²) cancels.Thus, E = (1 - t²)/(2 t )Similarly, from equation 1:S = (4 + D) t² = -t^4 - 2E tPlug E:(4 + D) t² = - t^4 - 2*( (1 - t²)/(2 t )) * tSimplify:(4 + D) t² = - t^4 - (1 - t² )=> (4 + D) t² = - t^4 - 1 + t²Bring all terms to left:(4 + D) t² + t^4 + 1 - t² = 0=> t^4 + (4 + D - 1) t² + 1 = 0=> t^4 + (3 + D) t² + 1 = 0But (4 + D) t² = - t^4 - (1 - t² )Wait, maybe better to solve for D:(4 + D) t² = - t^4 -1 + t²Then, 4 t² + D t² = - t^4 -1 + t²Bring all terms to left:D t² = - t^4 -1 + t² -4 t²=> D t² = - t^4 -1 -3 t²Thus,D = (- t^4 -1 -3 t² ) / t² = - t² - 3 - 1/t²So D = - (t² + 3 + 1/t² )Therefore, we have expressions for D and E in terms of t:D = - ( t² + 3 + 1/t² )E = (1 - t² )/(2 t )Thus, the equation of the circle is:x² + y² + D x + E y = 0Plug in D and E:x² + y² - ( t² + 3 + 1/t² ) x + [ (1 - t² )/(2 t ) ] y = 0Multiply through by 2 t to eliminate denominators:2 t x² + 2 t y² - 2 t ( t² + 3 + 1/t² ) x + (1 - t² ) y = 0Simplify each term:First term: 2 t x²Second term: 2 t y²Third term: -2 t ( t² + 3 + 1/t² ) x = -2 t^3 x - 6 t x - 2 x / tFourth term: (1 - t² ) ySo the equation becomes:2 t x² + 2 t y² - 2 t^3 x - 6 t x - (2 x)/t + (1 - t² ) y = 0Hmm, this seems messy. Maybe there's a better way to find the intersection point P.Alternatively, since P lies on both the circle and the parabola y² = 4x. Let me parametrize P as (s², 2s) on the parabola. Then substitute into the circle equation.So substituting x = s², y = 2s into the circle equation x² + y² + D x + E y = 0:(s²)^2 + (2s)^2 + D s² + E (2s) = 0=> s^4 + 4 s² + D s² + 2 E s = 0But this is similar to the equation we had for point A. Indeed, when s = t, we get the equation for point A, which is on the circle. Similarly, when s = -1/t, since point B is (1/t², -2/t), so s = -1/t. Thus, the equation s^4 + (4 + D) s² + 2 E s = 0 is satisfied by s = t and s = -1/t, which are points A and B. But we are looking for another intersection point P, so s ≠ t and s ≠ -1/t.Therefore, the equation s^4 + (4 + D) s² + 2 E s = 0 can be factored as (s - t)(s + 1/t)(quadratic) = 0. Let's check.Wait, the equation is quartic in s: s^4 + (4 + D)s² + 2 E s = 0. Since we know two roots s = t and s = -1/t (since points A and B are on both the parabola and the circle), so the equation can be written as (s - t)(s + 1/t)(s² + a s + b) = 0.Let me expand (s - t)(s + 1/t)(s² + a s + b):First multiply (s - t)(s + 1/t):= s² + (1/t)s - t s - t*(1/t)= s² + (1/t - t)s - 1Then multiply by (s² + a s + b):= (s² + (1/t - t)s -1)(s² + a s + b)Expanding:s^4 + a s^3 + b s² + (1/t - t)s^3 + a(1/t - t)s² + b(1/t - t)s - s² - a s - bCombine like terms:s^4 + [a + (1/t - t)] s^3 + [b + a(1/t - t) -1] s² + [b(1/t - t) - a] s - bCompare with original equation: s^4 + (4 + D)s² + 2 E s = 0. So coefficients:s^4: 1 (matches)s^3: [a + (1/t - t)] = 0 (since original equation has no s^3 term)s²: [b + a(1/t - t) -1] = (4 + D)s: [b(1/t - t) - a] = 2 Econstant term: -b = 0 => b = 0But wait, original equation has no constant term? Wait, original equation is s^4 + (4 + D)s² + 2 E s = 0. So constant term is 0. Hence, from our factorization, -b = 0 ⇒ b = 0.So with b=0, let's substitute back:From s^3 coefficient: a + (1/t - t) = 0 ⇒ a = t - 1/tFrom s² coefficient: [0 + a(1/t - t) -1] = (4 + D)Substitute a = t - 1/t:[ (t - 1/t)(1/t - t) - 1 ] = (4 + D)Compute (t - 1/t)(1/t - t):Multiply out:t*(1/t) - t^2 - (1/t^2) + 1= 1 - t² - 1/t² + 1= 2 - t² - 1/t²Thus:[2 - t² - 1/t² -1] = (4 + D)Simplify:(1 - t² - 1/t²) = 4 + D ⇒ D = 1 - t² - 1/t² -4 = -3 - t² -1/t²Which matches our previous expression for D. Good.From the s term coefficient: [0*(1/t - t) - a] = 2 E ⇒ -a = 2 E ⇒ E = -a/2 = -(t - 1/t)/2 = (1/t - t)/2, which also matches our previous expression for E.Therefore, the quartic equation factors as (s - t)(s + 1/t)(s² + a s + 0) = (s - t)(s + 1/t)(s² + (t - 1/t)s) = 0So the quadratic factor is s² + (t - 1/t)s = 0 ⇒ s(s + t - 1/t) = 0. So the roots are s=0 and s= (1/t - t). But s=0 corresponds to point O(0,0), which is already considered. The other root is s = (1/t - t). Therefore, the fourth intersection point P has parameter s = 1/t - t.Wait, so P is (s², 2s) where s = (1/t - t). Let's compute s:s = 1/t - t = (1 - t²)/tTherefore, coordinates of P:x = s² = [(1 - t²)/t]^2 = (1 - 2t² + t^4)/t²y = 2s = 2*(1 - t²)/tSo P( (1 - 2t² + t^4)/t² , 2(1 - t²)/t )Simplify x-coordinate:(1 - 2t² + t^4)/t² = (t^4 - 2t² +1)/t² = (t² -1)^2 / t²So P( (t² -1)^2 / t² , 2(1 - t²)/t )Alternatively, written as P( (t - 1/t)^2, -2(t - 1/t) )Wait, let's check:If s = (1/t - t), then y = 2s = 2(1/t - t) = 2/t - 2t. But from above, we had 2(1 - t²)/t = 2/t - 2t. So same thing.Similarly, x = s² = (1/t - t)^2 = 1/t² - 2 + t². Which is the same as (t² -1)^2 / t² = (t^4 - 2t² +1)/t² = t² - 2 + 1/t². So same as 1/t² -2 + t². So yes, same.So P is ( (1/t - t)^2, 2(1/t - t) )Alternatively, let me set u = t - 1/t, then P is (u², -2u). Because if u = t -1/t, then P's coordinates are (u², -2u). Hmm, interesting. So parametrizing P as (u², -2u). But (u², -2u) is a point on the parabola y² = 4x: (-2u)^2 = 4u² = 4x ⇒ x = u², which matches. So indeed, P can be written as (u², -2u) where u = t -1/t.But note that u can be any real number except zero, since t ≠ 0. Wait, but t can be any real number except zero, so u = t -1/t can take any real value except... Hmm, actually, u can be any real number except maybe some constraints? For example, if t >0, then u can be from (-infty, 0) when t approaches 0 from the positive side, to positive infinity as t approaches positive infinity. Similarly for t negative. So u can be any real number except perhaps zero? Wait, when t =1, u=0. But if t=1, then s =1/t - t=1 -1=0, so point P would be (0,0), which is O. But in the problem, P is different from O, A, B. So when s=0, it's O. So we need to exclude s=0, which corresponds to u=0. Thus, u ≠0. Therefore, P is (u², -2u) where u ≠0.But in our case, s =1/t -t, so u = t -1/t. So P is (u², -2u). Interesting. So parametrizing P in terms of u = t -1/t.But how does this help? Maybe not directly. Let's proceed.Now, we need to use the condition that PF bisects angle APB. So PF is the angle bisector of angle APB.So point P is on the parabola, and F is (1,0). So PF is the line from P(u², -2u) to F(1,0). We need this line to be the angle bisector of angle APB.Angle bisector condition: The angle between PF and PA should equal the angle between PF and PB.Alternatively, by the Angle Bisector Theorem, the ratio of the distances from a point on the angle bisector to the sides is equal to the ratio of the adjacent sides. But in this case, since PF bisects angle APB, the ratio PA/PB = FA/FB. Wait, maybe. Let me recall the Angle Bisector Theorem: In triangle APB, if PF is the angle bisector of angle APB, then PA / PB = AF / FB.But AF and FB are the lengths from A and B to F. Wait, but F is a fixed point (1,0). So need to compute AF and FB.Wait, points A(t², 2t) and F(1,0). Distance AF:√[(t² -1)^2 + (2t -0)^2] = √[(t^4 - 2t² +1) + 4t²] = √[t^4 + 2t² +1] = √[(t² +1)^2] = |t² +1| = t² +1 (since t² ≥0)Similarly, distance BF:Point B is (1/t², -2/t). Distance BF:√[(1/t² -1)^2 + (-2/t -0)^2] = √[(1 - 2/t² + 1/t^4) + 4/t²] = √[1/t^4 + 2/t² +1] = √[(1/t² +1)^2] = |1/t² +1| = 1/t² +1 (since 1/t² ≥0)Therefore, AF = t² +1, BF = 1 + 1/t². So AF / BF = (t² +1)/(1 + 1/t²) = (t² +1)/( (t² +1)/t² ) ) = t².Therefore, by the Angle Bisector Theorem, PA / PB = AF / FB = t².So PA / PB = t².Therefore, we have PA² / PB² = t^4.Compute PA² and PB².PA is the distance from P(u², -2u) to A(t², 2t):PA² = (u² - t²)^2 + (-2u - 2t)^2Similarly, PB² = (u² - 1/t²)^2 + (-2u + 2/t)^2But since u = t -1/t, let's substitute u into PA² and PB².First, compute u = t -1/t. Therefore, u + t = t -1/t + t = 2t -1/tSimilarly, u - (-1/t) = t -1/t +1/t = t.Wait, maybe better to express everything in terms of t.Let me compute PA²:PA² = (u² - t²)^2 + (-2u - 2t)^2But u = t -1/t, so:First term: u² - t² = (t -1/t)^2 - t² = t² - 2 + 1/t² - t² = -2 + 1/t²Second term: -2u -2t = -2(t -1/t) -2t = -2t + 2/t -2t = -4t + 2/tTherefore:PA² = (-2 + 1/t²)^2 + (-4t + 2/t)^2Compute first term squared:(-2 + 1/t²)^2 = 4 - 4/t² + 1/t^4Second term squared:(-4t + 2/t)^2 = 16 t² - 16 + 4/t²Therefore, PA² = 4 -4/t² +1/t^4 +16 t² -16 +4/t² = (4 -16) + (-4/t² +4/t²) +1/t^4 +16 t² = -12 +1/t^4 +16 t²Similarly, compute PB²:PB² = (u² -1/t²)^2 + (-2u +2/t)^2Again, u = t -1/t, so:First term: u² -1/t² = (t -1/t)^2 -1/t² = t² -2 +1/t² -1/t² = t² -2Second term: -2u +2/t = -2(t -1/t) +2/t = -2t +2/t +2/t = -2t +4/tTherefore:PB² = (t² -2)^2 + (-2t +4/t)^2First term squared:(t² -2)^2 = t^4 -4 t² +4Second term squared:(-2t +4/t)^2 =4 t² -16 +16/t²Thus, PB² = t^4 -4 t² +4 +4 t² -16 +16/t² = t^4 + ( -4t² +4t² ) + (4 -16) +16/t² = t^4 -12 +16/t²Therefore, PA² = 16 t² -12 +1/t^4PB² = t^4 -12 +16/t²Then, according to the Angle Bisector Theorem:PA² / PB² = (16 t² -12 +1/t^4 ) / (t^4 -12 +16/t² ) = t^4Wait, PA² / PB² = t^4.Therefore:(16 t² -12 +1/t^4 ) / (t^4 -12 +16/t² ) = t^4Multiply both sides by denominator:16 t² -12 +1/t^4 = t^4 (t^4 -12 +16/t² )Simplify RHS:t^4 * t^4 -12 t^4 +16 t^4 / t² = t^8 -12 t^4 +16 t²So equation becomes:16 t² -12 +1/t^4 = t^8 -12 t^4 +16 t²Subtract 16 t² -12 +1/t^4 from both sides:0 = t^8 -12 t^4 +16 t² -16 t² +12 -1/t^4Simplify:0 = t^8 -12 t^4 +12 -1/t^4Multiply both sides by t^4 to eliminate denominator:0 = t^12 -12 t^8 +12 t^4 -1So we get the equation:t^12 -12 t^8 +12 t^4 -1 = 0This looks daunting. Let me make substitution z = t^4. Then equation becomes:z^3 -12 z^2 +12 z -1 =0So need to solve cubic equation z³ -12 z² +12 z -1 =0.Let me try rational roots. Possible rational roots are ±1.Test z=1: 1 -12 +12 -1=0. Yes! z=1 is a root.Therefore, factor out (z -1):Divide z³ -12 z² +12 z -1 by (z -1).Using polynomial division or synthetic division:Coefficients: 1 | -12 | 12 | -1Divide by z -1:Bring down 1.Multiply by 1: 1.Add to next coefficient: -12 +1= -11.Multiply by1: -11.Add to next coefficient:12 + (-11)=1.Multiply by1:1.Add to last coefficient: -1 +1=0.So the cubic factors as (z -1)(z² -11 z +1)=0.Thus, roots are z=1, and roots of z² -11 z +1=0.Solve z² -11 z +1=0:z = [11 ±√(121 -4)]/2 = [11 ±√117]/2 = [11 ± 3√13]/2.Therefore, z=1, z=(11 +3√13)/2, z=(11 -3√13)/2.But z= t^4 ≥0. All roots are positive? Let's check:For z=(11 +3√13)/2: 3√13 ≈3*3.605≈10.815, so 11 +10.815≈21.815, divided by 2≈10.9075>0.For z=(11 -3√13)/2: 11 -10.815≈0.185>0. So all z roots are positive. Therefore, t^4=1, t^4=(11 +3√13)/2, or t^4=(11 -3√13)/2.Case 1: z=1 ⇒ t^4=1 ⇒ t²=1 ⇒ t=±1.Case 2: z=(11 +3√13)/2 ⇒ t^4=(11 +3√13)/2. Then t²= sqrt[(11 +3√13)/2]. Let me compute sqrt[(11 +3√13)/2]. Let me check if this is a perfect square or something, but perhaps not. Similarly for Case 3.But let's first check case 1: t=±1.If t=1, then t₁=1, t₂=-1/t=-1.So points A(1, 2), B(1, -2). Wait, but chord AB would be from (1,2) to (1,-2), which is the latus rectum. Then the circumcircle of triangle AOB. Points O(0,0), A(1,2), B(1,-2).The circumcircle of triangle AOB. Let's compute it.Points O(0,0), A(1,2), B(1,-2).The circle passing through these points. Let's find its equation.General equation x² + y² + Dx + Ey + F=0. Since O is on it, F=0.Plug in A(1,2): 1 +4 + D*1 + E*2=0 ⇒5 + D +2E=0Plug in B(1,-2):1 +4 +D*1 + E*(-2)=0 ⇒5 +D -2E=0So we have two equations:5 + D +2E=05 + D -2E=0Subtract the second equation from the first: 4E=0 ⇒E=0.Then, 5 + D=0 ⇒ D= -5.So equation is x² + y² -5x=0. Complete the square:x² -5x + y²=0 ⇒(x -5/2)^2 + y² = (25/4)So center at (5/2,0), radius 5/2.Find intersection with parabola y²=4x. Substitute y²=4x into circle equation:x² +4x -5x=0 ⇒x² -x=0 ⇒x(x -1)=0 ⇒x=0 or x=1. So points O(0,0) and A,B at x=1. But P is supposed to be different from O,A,B. So in this case, the circle only intersects the parabola at O,A,B, so no such point P exists. Therefore, t=1 is invalid? Contradiction.Wait, but when t=1, the circle intersects the parabola only at O,A,B, so there's no other point P. But the problem states that P is different from O,A,B, so this case is impossible. Therefore, t=1 is invalid. Similarly, t=-1 would lead to the same problem.Therefore, case 1 gives no solution. So we must consider cases 2 and 3.Case 2: z=(11 +3√13)/2. So t^4=(11 +3√13)/2. Then t²= sqrt[(11 +3√13)/2], so t can be real or imaginary? Wait, t is real, since we are working in the real plane.Thus, t² is positive, so t is real. So possible real values of t. Similarly for case 3.Case 3: z=(11 -3√13)/2. Let's compute this value numerically to check if it's positive.11 -3√13 ≈11 -3*3.605≈11 -10.815≈0.185>0. So z=0.185, so t^4≈0.185, so t²≈sqrt(0.185)≈0.43, so t≈±0.656.So cases 2 and 3 are valid.Thus, in total, we have three possible roots, but t=±1 lead to no point P, so only the other two roots are valid.Therefore, we need to consider t such that t^4=(11 +3√13)/2 or t^4=(11 -3√13)/2.But how does this relate to |PF|?We need to compute |PF|, the distance from P to F(1,0).Point P is ( (t² -1)^2 / t² , 2(1 - t²)/t )Alternatively, in terms of u = t -1/t, P is (u², -2u). So coordinates are (u², -2u). Then F is (1,0).So distance PF is sqrt[(u² -1)^2 + (-2u -0)^2] = sqrt[(u² -1)^2 +4u²] = sqrt[u^4 -2u² +1 +4u²] = sqrt[u^4 +2u² +1] = sqrt[(u² +1)^2] = |u² +1| = u² +1, since u² +1 is always positive.Therefore, |PF|=u² +1.But u = t -1/t, so u² = t² -2 +1/t². Therefore, |PF| = t² -2 +1/t² +1 = t² +1/t² -1.Thus, |PF|= t² +1/t² -1.But in the cases where t^4=(11 ±3√13)/2, we need to compute t² +1/t².Let me denote S = t² +1/t². Then since t^4 +1/t^4 = (t² +1/t²)^2 -2 = S² -2.Given z = t^4, so 1/z =1/t^4.So for each case:Case 2: z=(11 +3√13)/2. Then 1/z =2/(11 +3√13). Rationalize denominator:Multiply numerator and denominator by (11 -3√13):1/z = [2(11 -3√13)] / [ (11)^2 - (3√13)^2 ] = [22 -6√13]/[121 -9*13] = [22 -6√13]/[121 -117] = [22 -6√13]/4 = [11 -3√13]/2.So in case 2: z +1/z = (11 +3√13)/2 + (11 -3√13)/2 = (11 +3√13 +11 -3√13)/2 =22/2=11.Similarly, in case 3: z=(11 -3√13)/2, then 1/z =2/(11 -3√13). Rationalize:1/z= [2(11 +3√13)]/[ (11)^2 - (3√13)^2 ]= [22 +6√13]/[121 -117]= [22 +6√13]/4= [11 +3√13]/2.Then z +1/z= (11 -3√13)/2 + (11 +3√13)/2= (11 -3√13 +11 +3√13)/2=22/2=11.Therefore, in both cases, z +1/z=11.But S = t² +1/t². Since z = t^4, then S² = (t² +1/t²)^2 = t^4 +2 +1/t^4 = z +2 +1/z = (z +1/z) +2 =11 +2=13. Therefore, S²=13 ⇒ S=√13 or -√13. But S=t² +1/t² ≥2 (by AM ≥GM), so S=√13.Wait, but wait. Wait S²=13 ⇒ S=√13≈3.605, which is greater than 2, which is consistent.Therefore, in both cases, S=√13.Thus, |PF|= S -1=√13 -1.Therefore, regardless of the case, |PF|=√13 -1.Wait, but hold on. Wait, S = t² +1/t²=√13. Therefore, |PF|=√13 -1.Therefore, the possible value of |PF| is √13 -1.Therefore, the answer is boxed{sqrt{13} - 1}Final AnswerThe possible value of (|PF|) is (boxed{sqrt{13} - 1}).
✅:Given a parabola ( y^2 = 4x ) with a chord ( AB ) passing through the focus ( F(1,0) ), we need to find the possible values of ( |PF| ) where ( P ) is the intersection of the circumcircle of ( triangle AOB ) (other than ( O, A, B )) and the parabola, such that ( PF ) bisects ( angle APB ).1. Parametrization of Points ( A ) and ( B ): - Points ( A ) and ( B ) on the parabola can be parametrized as ( A(t^2, 2t) ) and ( B(1/t^2, -2/t) ) since the product of their parameters ( t_1 t_2 = -1 ).2. Circumcircle of ( triangle AOB ): - The circumcircle passes through ( O(0,0) ), ( A(t^2, 2t) ), and ( B(1/t^2, -2/t) ). The equation of the circle is derived and found to intersect the parabola at another point ( P ).3. Finding Point ( P ): - Using parametric coordinates, ( P ) is found to be ( left( left( t - frac{1}{t} right)^2, -2 left( t - frac{1}{t} right) right) ).4. Angle Bisector Condition: - Using the Angle Bisector Theorem, the ratio ( frac{PA}{PB} = frac{AF}{FB} ) is derived. This leads to solving a cubic equation in terms of ( t ).5. Solving the Cubic Equation: - The cubic equation ( z^3 - 12z^2 + 12z - 1 = 0 ) (where ( z = t^4 )) is solved. The roots are ( z = 1 ), ( z = frac{11 + 3sqrt{13}}{2} ), and ( z = frac{11 - 3sqrt{13}}{2} ).6. Calculating ( |PF| ): - For valid roots, ( |PF| ) is calculated as ( t^2 + frac{1}{t^2} - 1 ). This simplifies to ( sqrt{13} - 1 ).Thus, the possible value of ( |PF| ) is (boxed{sqrt{13} - 1}).
🔑:Given the parabola ( y^2 = 4x ) and a focal point ( F(1,0) ), we need to find all possible values of ( |PF| ) where ( P ) lies on the circumcircle of ( triangle AOB ) and ( PF ) bisects the angle ( angle APB ).1. Identify points on the parabola: Let ( Aleft(frac{y_1^2}{4}, y_1right) ), ( Bleft(frac{y_2^2}{4}, y_2right) ), and ( Pleft(frac{y_3^2}{4}, y_3right) ) be points on the parabola. By the properties of the parabola, we know ( y_1 ), ( y_2 ), and ( y_3 ) are distinct and non-zero.2. Equation of the chord ( AB ): The line ( AB ) has an equation of the form ( x = t y + 1 ). Substituting into the parabola equation, we get: [ y^2 = 4(t y + 1) ] [ y^2 - 4t y - 4 = 0 ] From the quadratic equation, the product of the roots is given by: [ y_1 y_2 = -4 ]3. Equation of the circumcircle of ( triangle AOB ): Since the circumcircle passes through the origin ( O ), let its general equation be: [ x^2 + y^2 + d x + e y = 0 ] When substituting ( x = frac{y^2}{4} ): [ frac{y^4}{16} + left(1 + frac{d}{4}right)y^2 + e y = 0 ] This quartic equation must have roots ( y_1, y_2, y_3, 0 ). By Vieta’s formulas, we know: [ y_1 + y_2 + y_3 + 0 = 0 implies y_3 = - (y_1 + y_2) ]4. Using the angle bisector theorem: Given ( PF ) bisects ( angle APB ), by the Angle Bisector Theorem: [ frac{|PA|}{|PB|} = frac{|FA|}{|FB|} ] [ frac{|y_1|}{|y_2|} = frac{|FA|}{|FB|} ] Calculating the distances: [ |PA| = sqrt{left(frac{y_3^2}{4} - frac{y_1^2}{4}right)^2 + (y_3 - y_1)^2} ] [ |PB| = sqrt{left(frac{y_3^2}{4} - frac{y_2^2}{4}right)^2 + (y_3 - y_2)^2} ]5. Expressing the ratio: [ frac{y_1^2}{y_2^2} = frac{|PA|^2}{|PB|^2} = frac{left(frac{(y_1 + y_2)^2 - y_1^2}{4}right)^2 + (2y_1 + y_2)^2}{left(frac{(y_1 + y_2)^2 - y_2^2}{4}right)^2 + (y_1 + 2y_2)^2} ] Simplifying yields to: [ frac{(y_2^2 - 8)^2 + 16(4y_1^2 + y_2^2 - 16)}{(y_1^2 - 8)^2 + 16(4y_2^2 + y_1^2 - 16)} = frac{y_2^4 + 64y_1^2 - 192}{y_1^4 + 64y_2^2 - 192} ]6. Equating numerator and denominator: [ y_1^6 + 64 y_1^2 y_2^2 - 192 y_1^2 = y_2^6 + 64 y_2^2 y_1^2 - 192 y_2^2 ] Therefore: [ (y_1^2 - y_2^2)(y_1^4 + y_1^2 y_2^2 + y_2^4 - 192) = 0 ]7. Solving the factorized equation: - If ( y_1^2 = y_2^2 ), then ( y_2 = -y_1 ): [ y_3 = 0 ] This implies point ( P ) coincides with ( O ), which is not the case. Thus, this solution is invalid. - If ( y_1^4 + y_1^2 y_2^2 + y_2^4 = 192 ): [ (y_1^2 + y_2^2)^2 = 192 + (-4)^2 = 192 + 16 = 208 ] [ y_1^2 + y_2^2 = 4sqrt{13} ]8. Calculating ( |PF| ): [ |PF| = frac{y_3^2}{4} + 1 = frac{(y_1 + y_2)^2 + 4}{4} = frac{y_1^2 + y_2^2 - 4}{4} = frac{4sqrt{13} - 4}{4} = sqrt{13} - 1 ] Conclusion:[boxed{sqrt{13} - 1}]
❓:The Wolf and Ivan Tsarevich are 20 versts away from a source of living water, and the Wolf is taking Ivan Tsarevich there at a speed of 3 versts per hour. To revive Ivan Tsarevich, one liter of water is needed, which flows from the source at a rate of half a liter per hour. At the source, there is a Raven with unlimited carrying capacity; it must gather the water, after which it will fly towards the Wolf and Ivan Tsarevich at a speed of 6 versts per hour, spilling a quarter liter of water every hour. After how many hours will it be possible to revive Ivan Tsarevich?
🧠:Okay, so there's this problem about the Wolf, Ivan Tsarevich, and the Raven. Let me try to understand what's going on here. First, the Wolf and Ivan are 20 versts away from the source of living water. The Wolf is taking Ivan there at a speed of 3 versts per hour. So, they're moving towards the source, right? Wait, no—if they're 20 versts away from the source, and the Wolf is taking Ivan there, that means they're moving towards the source at 3 versts per hour. So, time taken for them to reach the source would be distance divided by speed, which is 20 / 3 hours, approximately 6.666... hours. But maybe we don't need that yet.But the problem isn't just about them reaching the source. The key is that Ivan needs to be revived with one liter of water. The source produces water at half a liter per hour. The Raven is at the source and can carry unlimited water. So, the Raven needs to collect water from the source and then fly to meet the Wolf and Ivan, who are moving towards the source. However, the Raven spills a quarter liter of water every hour while flying. So, the Raven has to gather some amount of water, then fly towards the moving target (Wolf and Ivan) and deliver at least one liter without it all spilling out. The question is: After how many hours will it be possible to revive Ivan Tsarevich?Hmm. So, the process would be: the Raven starts collecting water at the source. At some point, the Raven decides to stop collecting and start flying towards the Wolf and Ivan. But the Wolf and Ivan are getting closer to the source over time. So, the Raven's flight time will depend on when it departs. The later the Raven departs, the closer the Wolf and Ivan are, so the shorter the flight time, but the more water the Raven could have collected. However, the Raven spills water during the flight. So, the total water delivered is the amount collected minus the spillage during flight. We need this delivered water to be at least 1 liter.Let me structure the problem.Let’s denote t as the time in hours after which the Raven departs from the source. So, during time t, the Raven is collecting water at 0.5 liters per hour, so collected water is 0.5 * t liters.Then, the Raven starts flying towards the Wolf and Ivan. At that moment, the Wolf and Ivan have been traveling for t hours towards the source. Since their speed is 3 versts per hour, they have covered 3t versts, so the remaining distance to the source is 20 - 3t versts.The Raven's speed is 6 versts per hour, and the Wolf is moving towards the source at 3 versts per hour. So, their relative speed towards each other is 6 + 3 = 9 versts per hour. Therefore, the time it takes for the Raven to meet them is (20 - 3t) / 9 hours. Let's denote this flight time as f. So, f = (20 - 3t)/9.During this flight time, the Raven spills 0.25 liters per hour. So, total spillage is 0.25 * f liters. Therefore, the water delivered is 0.5t - 0.25f. This needs to be >= 1 liter.So, the equation is:0.5t - 0.25*( (20 - 3t)/9 ) >= 1Let me write that out:0.5t - 0.25*( (20 - 3t)/9 ) >= 1First, simplify this inequality. Let's convert 0.5 to 1/2 and 0.25 to 1/4 for easier calculation.(1/2)t - (1/4)*( (20 - 3t)/9 ) >= 1Multiply through by 36 to eliminate denominators:36*(1/2)t - 36*(1/4)*( (20 - 3t)/9 ) >= 36*1Simplify each term:18t - 9*( (20 - 3t)/9 ) >= 36Simplify the second term: 9*( (20 - 3t)/9 ) = (20 - 3t)So:18t - (20 - 3t) >= 36Combine like terms:18t -20 + 3t >= 3621t -20 >= 3621t >= 56t >= 56 /21t >= 8/3 ≈ 2.666... hoursSo, t must be at least 8/3 hours. But wait, is this the total time?Wait, t is the time the Raven spends collecting water before departing. Then, the flight time f is (20 - 3t)/9 hours. So, the total time until the Raven meets the Wolf and Ivan is t + f.So, the total time until Ivan can be revived is t + (20 - 3t)/9.Let me compute that:Total time = t + (20 - 3t)/9 = (9t +20 -3t)/9 = (6t +20)/9Given that t must be >= 8/3 hours, then the minimal total time would be when t is 8/3.Let me compute that:Total time = (6*(8/3) +20)/9 = (16 +20)/9 = 36/9 =4 hours.Wait, so total time is 4 hours? That seems interesting. Let me check the steps again.First, the Raven collects water for t hours, accumulating 0.5t liters.Then, flies for (20 -3t)/9 hours, spilling 0.25*(20 -3t)/9 liters.Thus, the delivered water is 0.5t -0.25*(20 -3t)/9 >=1.Solving gives t >=8/3.Then, total time is t + (20 -3t)/9. Let's substitute t=8/3:Total time=8/3 + (20 -3*(8/3))/9=8/3 + (20 -8)/9=8/3 +12/9=8/3 +4/3=12/3=4.Yes, that's 4 hours. So, the answer is 4 hours.But let me verify this with the numbers.If the Raven departs after 8/3 hours (~2.6667 hours):Water collected: 0.5*(8/3)=4/3 liters≈1.333 liters.Distance remaining for Wolf and Ivan:20 -3*(8/3)=20 -8=12 versts.Time to fly 12 versts at closing speed 9 versts/h:12/9=4/3 hours≈1.333 hours.Spillage during flight:0.25*(4/3)=1/3 liters≈0.333 liters.Water delivered:4/3 -1/3=3/3=1 liter. Exactly the amount needed. So that works.Total time:8/3 +4/3=12/3=4 hours. Perfect.But let me check if departing earlier would allow for a shorter total time. Suppose t is less than8/3. Then the flight time would be longer, but the collected water would be less. Let's try t=2 hours.Water collected:0.5*2=1 liter.Distance remaining:20 -3*2=14 versts.Flight time:14/9≈1.555 hours.Spillage:0.25*14/9≈0.388 liters.Water delivered:1 -0.388≈0.611 liters, which is less than 1. Not enough.If the Raven departs earlier, even though flight time is longer, the collected water isn't enough.Alternatively, if the Raven departs later, say t=3 hours.Water collected:1.5 liters.Distance remaining:20 -9=11 versts.Flight time:11/9≈1.222 hours.Spillage:0.25*11/9≈0.305 liters.Water delivered:1.5 -0.305≈1.194 liters. That's more than 1 liter, which is sufficient. But total time would be 3 +11/9≈3 +1.222=4.222 hours, which is more than 4. So departing later gives more total time but enough water. But since we want the earliest possible time, 4 hours is better.Therefore, the minimal total time is 4 hours.But another check: suppose the Raven doesn't wait at all, departs immediately at t=0.Water collected:0 liters. Useless.If the Raven departs at t=1 hour:Water collected:0.5 liters.Distance remaining:20 -3=17 versts.Flight time:17/9≈1.888 hours.Spillage:0.25*17/9≈0.472 liters.Water delivered:0.5 -0.472≈0.028 liters. Not enough.Similarly, t=2.5 hours:Water collected:1.25 liters.Distance remaining:20 -7.5=12.5 versts.Flight time:12.5/9≈1.388 hours.Spillage:0.25*1.388≈0.347 liters.Water delivered:1.25 -0.347≈0.903 liters. Still not enough.So, t must be at least8/3≈2.6667 hours. So, the minimal total time is indeed 4 hours.Therefore, the answer is 4 hours.Final Answerboxed{4}
✅:The Wolf and Ivan Tsarevich are 20 versts away from a source of living water, moving towards it at 3 versts per hour. The Raven needs to collect water from the source, which flows at 0.5 liters per hour, and then fly to meet them, spilling 0.25 liters per hour. The goal is to determine the minimum time required to deliver 1 liter of water to revive Ivan Tsarevich.1. Let ( t ) be the time (in hours) the Raven spends collecting water before departing.2. During this time, the Wolf and Ivan travel ( 3t ) versts, reducing their distance to the source to ( 20 - 3t ) versts.3. The Raven's flight time to meet them is ( frac{20 - 3t}{9} ) hours (since their relative speed is 9 versts per hour).4. Water collected by the Raven: ( 0.5t ) liters.5. Water spilled during flight: ( 0.25 times frac{20 - 3t}{9} ) liters.6. The equation for the water delivered is: [ 0.5t - 0.25 times frac{20 - 3t}{9} geq 1 ]7. Simplifying this inequality: [ frac{1}{2}t - frac{1}{4} times frac{20 - 3t}{9} geq 1 ] Multiplying through by 36: [ 18t - (20 - 3t) geq 36 ] Simplifying: [ 21t - 20 geq 36 implies 21t geq 56 implies t geq frac{8}{3} ]8. The total time is ( t + frac{20 - 3t}{9} ). Substituting ( t = frac{8}{3} ): [ text{Total time} = frac{8}{3} + frac{20 - 3 times frac{8}{3}}{9} = frac{8}{3} + frac{12}{9} = frac{8}{3} + frac{4}{3} = frac{12}{3} = 4 ]Thus, the minimum time required to revive Ivan Tsarevich is (boxed{4}) hours.
🔑:1. Determine the time the wolf will take to reach the source: Given the speed of the wolf is 3 versts per hour and the distance to the source is 20 versts: [ t_{text{wolf}} = frac{20}{3} text{ hours} ]2. Calculate the amount of time required to collect 1 liter of water: The water flows at a rate of (frac{1}{2}) liter per hour. To collect 1 liter: [ t_{text{collect}} = frac{1}{frac{1}{2}} = 2 text{ hours} ]3. Total time for both wolf and water collection: The wolf needs (frac{20}{3}) hours to reach the source and another 2 hours for the water to collect: [ t_{text{total}} = frac{20}{3} + 2 = frac{20}{3} + frac{6}{3} = frac{26}{3} text{ hours} ]4. Determine the average distance the raven will need to travel: The raven will meet the wolf and Ivan halfway. Therefore: [ text{distance}_{text{raven}} = frac{20}{2} = 10 text{ versts} ]5. Calculate the total time the raven takes to deliver the water: The raven flies at 6 versts per hour and loses (frac{1}{4}) liter of water per hour. To meet halfway: [ t_{text{raven}} = frac{10}{6} text{ hours} ] Calculate amount of water lost: [ text{water}_{text{lost}} = frac{t_{text{raven}}}{4} = frac{frac{10}{6}}{4} = frac{10}{24} = frac{5}{12} ] 6. Ensure there is sufficient water when the raven reaches Ivan and the wolf: The raven starts with 1 liter of water. After flying and losing some water: [ 1 - frac{5}{12} = frac{12}{12} - frac{5}{12} = frac{7}{12} text{ liters remaining} ]Since (frac{7}{12}) liters is more than zero, the raven can indeed deliver sufficient water for the reanimation of Ivan. Conclusion:[boxed{4 text{ hours}}]
❓:Compute the number of ordered pairs of positive integers ((a, b)) satisfying the equation (operatorname{gcd}(a, b) cdot a + b^2 = 10000).
🧠:Okay, let's try to figure out how to solve this problem: Find the number of ordered pairs of positive integers (a, b) such that gcd(a, b) * a + b² = 10000. Hmm, that looks a bit tricky, but let's break it down step by step.First, let's recall that gcd(a, b) stands for the greatest common divisor of a and b. So, maybe we can express a and b in terms of their gcd to simplify the equation. Let's let d = gcd(a, b). Then, we can write a = d * x and b = d * y, where x and y are positive integers that are coprime, meaning gcd(x, y) = 1. That makes sense because if there was a common divisor greater than 1, then d wouldn't be the greatest common divisor.Substituting these into the original equation: gcd(a, b) * a + b² = d * (d * x) + (d * y)² = d²x + d²y² = d²(x + y²) = 10000. So, we have d²(x + y²) = 10000. Since d² is a perfect square, it must divide 10000. Let's factor 10000 to see its square factors. 10000 = 10^4 = (2 * 5)^4 = 2^4 * 5^4. The square factors of 10000 are the squares of the divisors of 10^4. So, the exponents of 2 and 5 in d² must be even and less than or equal to 4. Therefore, possible exponents for 2 in d are 0, 2, 4 and for 5 are 0, 2, 4. Therefore, d can be 1, 2^2=4, 2^4=16, 5^2=25, 2^2*5^2=100, 2^4*5^2=400, 5^4=625, 2^2*5^4=2500, 2^4*5^4=10000. Wait, but 10000 is 10^4, so d² must divide 10^4, hence d must divide 10²=100. Wait, maybe that's a better way: since d² divides 10^4, then d divides 10²=100. Because if d² divides 10^4, then d divides 10². Let me check. Let’s say d² divides 10^4. Then d² | 10^4, so d must divide 10². Because 10^4 is (10²)^2, so the divisors of (10²)^2 squared are numbers whose square divides it, which is equivalent to the divisors of 10². Therefore, d can be any divisor of 100. Therefore, possible values of d are the divisors of 100. So, divisors of 100 are 1, 2, 4, 5, 10, 20, 25, 50, 100. Therefore, d can be any of these. So, there are 9 possible values for d? Wait, but earlier when I considered exponents, I had more possibilities, but maybe that's not correct. Let's check. If d divides 100, then d can be 1, 2, 4, 5, 10, 20, 25, 50, 100. So 9 divisors. Then d² would divide 100² = 10000. So yes, d can be each of these divisors. So, 9 possible d's.Okay, so for each d in {1, 2, 4, 5, 10, 20, 25, 50, 100}, we can compute x + y² = 10000 / d². Then, since a = d*x and b = d*y, and gcd(x, y) = 1, we need to find the number of coprime pairs (x, y) such that x + y² = N, where N = 10000 / d². Then, for each d, compute N, then find the number of (x, y) with x = N - y², x positive integer, and gcd(x, y) = 1. Then, sum over all d.So the plan is:1. Enumerate all possible d (divisors of 100).2. For each d, compute N = 10000 / d².3. For each N, find the number of y such that y is a positive integer, y² < N (since x must be positive), and gcd(N - y², y) = 1.4. Sum these counts over all d.Let me verify that. Since x must be a positive integer, N - y² must be positive, so y² < N. Therefore, y must be less than sqrt(N). So for each d, we need to find all y in [1, floor(sqrt(N - 1))] such that gcd(N - y², y) = 1. Then, the number of such y is the count for that d.But since x and y must be coprime, and x = N - y², so we need gcd(N - y², y) = 1.Alternatively, since x = N - y², and we need gcd(x, y) = 1. So yes, the condition is gcd(N - y², y) = 1.Therefore, for each d, we compute N = 10000 / d², then for each y from 1 to floor(sqrt(N - 1)), check if gcd(N - y², y) = 1, and count those y.So let's proceed step by step.First, list all divisors d of 100:1, 2, 4, 5, 10, 20, 25, 50, 100.For each d, compute N:d=1: N=10000/1=10000d=2: N=10000/4=2500d=4: N=10000/16=625d=5: N=10000/25=400d=10: N=10000/100=100d=20: N=10000/400=25d=25: N=10000/625=16d=50: N=10000/2500=4d=100: N=10000/10000=1Now, for each N, we need to find y such that y² < N and gcd(N - y², y) = 1.Let's handle each case:Case 1: d=1, N=10000We need y in [1, floor(sqrt(9999))] = [1, 99]For each y from 1 to 99, check if gcd(10000 - y², y) = 1.But 10000 - y² = (100 - y)(100 + y) + y²? Wait, no, 10000 - y² is just (100 - y)(100 + y). Wait, 100² = 10000, so 10000 - y² = (100 - y)(100 + y). So, for each y from 1 to 99, we have x = (100 - y)(100 + y). Then, we need gcd(x, y) = 1.But x = (100 - y)(100 + y). So, if y shares a common divisor with x, which is (100 - y)(100 + y), then that divisor must divide both y and (100 - y)(100 + y). Let's let’s denote d = gcd(y, (100 - y)(100 + y)). Since y and 100 ± y are related. Note that gcd(y, 100 - y) divides 100, because gcd(y, 100 - y) divides y + (100 - y) = 100. Similarly, gcd(y, 100 + y) divides (100 + y) - y = 100. So, gcd(y, 100 - y) divides 100 and y, and gcd(y, 100 + y) divides 100 and y. Therefore, any common divisor of y and x must divide 100 and y. So, to have gcd(x, y) = 1, we need that y divides 100 and also that y is coprime to 100. Wait, maybe not exactly. Let me think again.Suppose d divides y and (100 - y)(100 + y). Then, d divides y, so d divides 100 - y if and only if d divides 100, because 100 - y + y = 100. Similarly, d divides 100 + y - y = 100. So, d divides 100. Therefore, gcd(y, (100 - y)(100 + y)) divides 100. Therefore, to have gcd(y, x) = 1, we need that gcd(y, 100) = 1. Because if d divides y and 100, then d divides x. Therefore, if y is coprime to 100, then gcd(y, x) = 1.Wait, let me check. Suppose y and 100 are coprime. Then, does that ensure that gcd(y, x) = 1?Since x = 10000 - y² = (100 - y)(100 + y). If y is coprime to 100, then y is coprime to both 100 - y and 100 + y?Wait, not necessarily. For example, take y=3, which is coprime to 100. Then, 100 - 3 = 97, 100 + 3 = 103. Both primes, so gcd(3, 97*103) = gcd(3, 97*103) = 1, since 3 doesn't divide 97 or 103. Similarly, y=7: 100 -7=93, 100 +7=107. 93=3*31, so gcd(7, 93*107)=gcd(7,3*31*107)=1, since 7 doesn't divide any of those factors.But wait, what if y is coprime to 100 but shares a factor with 100 - y or 100 + y? Let's take y=21, which is coprime to 100. Then 100 -21=79, 100 +21=121. gcd(21,79*121)=gcd(21,79*121). 21=3*7. 79 is prime, 121=11². So gcd(21,79*121)=1, since 3,7 don't divide 79 or 121. So yes, seems like if y is coprime to 100, then gcd(y, (100 - y)(100 + y))=1. Therefore, in this case, the number of y from 1 to 99 with gcd(y, 100) = 1 would be the count. Wait, but 100=2²*5², so the number of integers from 1 to 99 coprime to 100 is φ(100)=100*(1-1/2)*(1-1/5)=40. But since y is up to 99, which is less than 100, φ(100) counts numbers from 1 to 100, so subtract 1 if 100 is included. But 100 is not included here, so φ(100) is 40, but since we're only up to 99, which is 1 less than 100. But 100 is not coprime to itself, so actually, numbers from 1 to 99 coprime to 100 are exactly φ(100)=40. So there are 40 numbers y between 1 and 99 that are coprime to 100. Therefore, does this imply that all these y satisfy gcd(y, x)=1? From the above examples, it seems so. Therefore, maybe for d=1, the count is 40.But wait, let's check a case where y is not coprime to 100 but still gcd(y, x) = 1. For example, take y=2, which is not coprime to 100. Then x = 10000 - 4 = 9996. gcd(2, 9996)=2, which is not 1. So, indeed, if y is not coprime to 100, then gcd(y, x) is at least 2. Therefore, only the y coprime to 100 will satisfy gcd(x, y)=1. Hence, for d=1, the number of solutions is φ(100)=40.But wait, hold on. Let's check another y not coprime to 100. Let's take y=5. Then x=10000 -25=9975. gcd(5,9975)=5, which is not 1. So, again, if y shares a factor with 100, then x will also share that factor, leading to gcd(y, x) >1. Hence, indeed, the only y that work are those coprime to 100, so 40 solutions.Therefore, for d=1, count is 40.Case 2: d=2, N=2500So, x + y² = 2500, with x=2500 - y², and gcd(x, y) =1. Since a=2x, b=2y, so x and y must be positive integers with x=2500 - y² >0, so y < sqrt(2500)=50. So y from 1 to 49.We need to count y in [1,49] such that gcd(2500 - y², y) =1.Again, let's analyze this. Let’s denote x = 2500 - y². Then, gcd(x, y) = gcd(2500 - y², y). Let's note that gcd(2500 - y², y) = gcd(2500, y). Because:gcd(2500 - y², y) = gcd(2500, y). Because:gcd(2500 - y², y) = gcd(2500, y), since gcd(a - b, b) = gcd(a, b). Here, a=2500, b=y². So, gcd(2500 - y², y) = gcd(2500, y², y). But gcd(2500, y², y) = gcd(2500, y). Because y² and y share all prime factors of y. Therefore, gcd(2500, y², y) = gcd(2500, y).Therefore, gcd(x, y) = gcd(2500, y). Therefore, to have gcd(x, y)=1, we need gcd(2500, y)=1. So y must be coprime to 2500. Since 2500=2²*5⁴, so y must not be divisible by 2 or 5. Therefore, the number of such y in [1,49] is equal to φ(2500) scaled down? Wait, no. The number of integers from 1 to 49 coprime to 2500 is equal to the number of integers from 1 to 49 not divisible by 2 or 5.Let's compute this:Total numbers: 49Numbers divisible by 2: floor(49/2)=24Numbers divisible by 5: floor(49/5)=9Numbers divisible by both 2 and 5: floor(49/10)=4Therefore, using inclusion-exclusion:Numbers divisible by 2 or 5: 24 + 9 - 4 = 29Therefore, numbers coprime to 2500: 49 -29=20Therefore, for d=2, count is 20.Case 3: d=4, N=625x + y² =625, y < sqrt(625)=25, so y from 1 to 24.gcd(625 - y², y) =1.Again, similar to previous cases:gcd(625 - y², y) = gcd(625, y). Because:gcd(625 - y², y) = gcd(625, y², y) = gcd(625, y). Therefore, to have gcd(x, y)=1, we need gcd(625, y)=1. 625=5⁴. Therefore, y must not be divisible by 5.Number of y from 1 to 24 not divisible by 5: total 24, subtract those divisible by 5. Numbers divisible by 5: floor(24/5)=4 (5,10,15,20). So 24 -4=20. Therefore, count=20.But wait, let's verify with inclusion-exclusion:Total y:24Divisible by 5:4So coprime:24 -4=20. Yes.But wait, let's check with actual gcd(625, y). Since 625=5⁴, so any y not divisible by 5 is coprime to 625. Therefore, count is 20.Therefore, for d=4, count=20.Case 4: d=5, N=400x + y² =400, y < sqrt(400)=20, so y from 1 to19.gcd(400 - y², y)=1.Again, compute gcd(400 - y², y) = gcd(400, y). Because:gcd(400 - y², y) = gcd(400, y², y) = gcd(400, y).Therefore, to have gcd(x, y)=1, need gcd(400, y)=1. 400=2^4*5^2. Therefore, y must be coprime to 400, i.e., not divisible by 2 or 5.Number of y from 1 to19 not divisible by 2 or 5:Total numbers:19Numbers divisible by 2: floor(19/2)=9Numbers divisible by5: floor(19/5)=3Numbers divisible by10: floor(19/10)=1So numbers divisible by 2 or5:9 +3 -1=11Thus, numbers coprime:19 -11=8Therefore, count=8.Case 5: d=10, N=100x + y²=100, y <10, so y from1 to9.gcd(100 - y², y)=1.Again, compute gcd(100 - y², y)=gcd(100, y). Because:gcd(100 - y², y)=gcd(100, y², y)=gcd(100, y).Therefore, need gcd(100, y)=1. 100=2²*5². So y must be coprime to 100, i.e., not divisible by 2 or 5.Numbers from1 to9:Total=9Divisible by2:4 (2,4,6,8)Divisible by5:1 (5)Divisible by10:0So numbers divisible by2 or5:4+1-0=5Numbers coprime:9-5=4Therefore, count=4.Case6: d=20, N=25x + y²=25, y <5, so y from1 to4.gcd(25 - y², y)=1.Compute gcd(25 - y², y)=gcd(25, y). Because:gcd(25 - y², y) = gcd(25, y², y)=gcd(25, y).Therefore, need gcd(25, y)=1. 25=5². So y must not be divisible by5.But y ranges from1 to4, none of which are divisible by5. Therefore, all y=1,2,3,4 satisfy gcd(25, y)=1. Therefore, count=4.Wait, but let's check:For each y in 1-4:y=1: x=25 -1=24, gcd(24,1)=1? Yes.Wait, but wait, gcd(24,1)=1, yes.Wait, but according to our earlier logic, gcd(x, y)=gcd(24,1)=1. So that's valid.Wait, but according to our formula, since gcd(25, y)=gcd(25,1)=1, so yes.But y=2: x=25 -4=21. gcd(21,2)=1.Similarly, y=3: x=25 -9=16. gcd(16,3)=1.y=4: x=25 -16=9. gcd(9,4)=1.So all 4 pairs are valid. Therefore, count=4.Case7: d=25, N=16x + y²=16, y <4, so y from1 to3.gcd(16 - y², y)=1.Compute gcd(16 - y², y)=gcd(16, y). Because:gcd(16 - y², y) = gcd(16, y², y)=gcd(16, y).So need gcd(16, y)=1. 16=2⁴, so y must be odd.y from1 to3: y=1,3.So count=2.Check each:y=1: x=16-1=15. gcd(15,1)=1. Yes.y=3: x=16-9=7. gcd(7,3)=1. Yes.y=2: x=16-4=12. gcd(12,2)=2≠1. So excluded. So indeed count=2.Case8: d=50, N=4x + y²=4, y <2, so y=1.gcd(4 -1=3,1)=1. So yes.Therefore, count=1.Case9: d=100, N=1x + y²=1. Since x and y are positive integers, y must be at least1, so y²=1 implies x=0, which is not positive. Therefore, no solutions here. Count=0.Now, summing up all counts:d=1:40d=2:20d=4:20d=5:8d=10:4d=20:4d=25:2d=50:1d=100:0Total=40+20+20+8+4+4+2+1+0= 40+20=60; 60+20=80; 80+8=88; 88+4=92; 92+4=96; 96+2=98; 98+1=99; 99+0=99.Wait, so total number of ordered pairs is 99? Hmm, that seems low? Let me check the calculations again.Wait, let's re-add:40 (d=1)+20 (d=2)=60+20 (d=4)=80+8 (d=5)=88+4 (d=10)=92+4 (d=20)=96+2 (d=25)=98+1 (d=50)=99+0=99. Yes, total 99.But let's verify one of the cases to be sure. Let's take d=25, N=16, which we said had 2 solutions. The pairs would be (a,b)=(25x,25y). x=15, y=1 gives a=375, b=25; x=7, y=3 gives a=175, b=75. Then, gcd(375,25)=25, so gcd(375,25)*375 +25²=25*375 +625=9375 +625=10000. Correct. Similarly, gcd(175,75)=25, 25*175 +75²=4375 +5625=10000. Correct.Another check: d=50, N=4, count=1. Then y=1, x=4 -1=3. So a=50*3=150, b=50*1=50. gcd(150,50)=50. Then, 50*150 +50²=7500 +2500=10000. Correct.Another check: d=20, N=25, y=1 to4, count=4. For example, y=1: x=24, a=20*24=480, b=20*1=20. gcd(480,20)=20. Then 20*480 +20²=9600 +400=10000. Correct. Similarly, y=2: x=21, a=420, b=40. gcd(420,40)=20. 20*420 +1600=8400 +1600=10000. Correct.Seems like the counts per d are correct. So total 99 ordered pairs.But wait, let's check the case when d=1, count=40. For example, take y=1, x=9999. Then a=1*9999, b=1*1. gcd(9999,1)=1. Then 1*9999 +1=10000. Correct. Similarly, y=99: x=10000 -99²=10000 -9801=199. a=199, b=99. gcd(199,99)=1. Then 1*199 +99²=199 +9801=10000. Correct.Therefore, seems like all counts are correct, so total 99 ordered pairs. Therefore, the answer is 99. Wait, but the problem says "ordered pairs of positive integers (a, b)", so we need to check if we have counted all possible a and b correctly.But according to our method, for each d (divisor of 100), we found the number of coprime pairs (x, y) such that x + y²=10000/d², then a=dx, b=dy. Since each (x, y) gives a unique (a,b) via a=dx, b=dy, and different d's will lead to different (a, b) pairs because d is the gcd(a, b). Therefore, there's no overlap between the pairs generated by different d's. Therefore, summing them gives the total number of solutions. Therefore, the total is 99.But let me check if there's a mistake in the logic. For example, when we set d = gcd(a, b), and then write a=dx, b=dy, with gcd(x, y)=1, this is a standard approach. Then, substituting into the equation, we get d²(x + y²)=10000. Then, since d² divides 10000, d must divide 100. Then, for each d, we compute N=10000/d², and find the number of (x, y) with x + y²=N and gcd(x, y)=1. Which is equivalent to counting y such that y < sqrt(N), and gcd(N - y², y)=1. Then, via the reasoning, we found that in each case, the count is equal to the number of y coprime to some factor related to N. For instance, for d=1, N=10000, which factors into 2^4*5^4, so y must be coprime to 100. For d=2, N=2500=2²*5⁴, so y must be coprime to 2500, which is equivalent to coprime to 2 and 5. But since y is up to 49, we counted those not divisible by 2 or5. Similarly for others. So the logic seems correct.Therefore, the total number is 99. But let's check online or see if another approach gives the same result. Alternatively, perhaps there's an error in the inclusion-exclusion counts.For example, for d=2, N=2500, y from1 to49. Numbers coprime to 2500 (i.e., not divisible by 2 or5). Total numbers:49.Numbers divisible by2: floor(49/2)=24 (since 2*24=48 <49)Numbers divisible by5: floor(49/5)=9 (5*9=45 <49)Numbers divisible by10: floor(49/10)=4 (10*4=40 <49)So numbers divisible by2 or5:24 +9 -4=29Thus, numbers coprime:49-29=20. Correct.Similarly, for d=5, N=400, y from1 to19. Coprime to400 (not divisible by2 or5). Numbers:Total=19Divisible by2:9 (2,4,...,18)Divisible by5:3 (5,10,15)Divisible by10:1 (10)So numbers divisible by2 or5:9 +3 -1=11Numbers coprime:19-11=8. Correct.Other cases checked similarly. Hence, the total is indeed 99. Therefore, the answer should be boxed{99}.
✅:To compute the number of ordered pairs of positive integers ((a, b)) satisfying (operatorname{gcd}(a, b) cdot a + b^2 = 10000), we start by expressing (a) and (b) in terms of their greatest common divisor (gcd). Let (d = operatorname{gcd}(a, b)), then (a = d cdot x) and (b = d cdot y) where (operatorname{gcd}(x, y) = 1). Substituting these into the equation gives:[d^2(x + y^2) = 10000]We need to find all divisors (d) of 100 (since (d^2) must divide 10000). The divisors of 100 are (1, 2, 4, 5, 10, 20, 25, 50, 100). For each (d), we compute (N = frac{10000}{d^2}) and find the number of coprime pairs ((x, y)) such that (x + y^2 = N). Key Steps for Each (d):1. Calculate (N): For each divisor (d), compute (N = frac{10000}{d^2}).2. Determine Valid (y): For each (N), find (y) such that (y^2 < N) and (operatorname{gcd}(N - y^2, y) = 1).3. Count Valid Pairs: The number of valid (y) values for each (d) is calculated using inclusion-exclusion principles and properties of gcd. Detailed Calculations:- (d = 1): (N = 10000). Valid (y) values are coprime to 100. Count: (phi(100) = 40).- (d = 2): (N = 2500). Valid (y) values are coprime to 2500. Count: 20.- (d = 4): (N = 625). Valid (y) values are coprime to 625. Count: 20.- (d = 5): (N = 400). Valid (y) values are coprime to 400. Count: 8.- (d = 10): (N = 100). Valid (y) values are coprime to 100. Count: 4.- (d = 20): (N = 25). Valid (y) values are coprime to 25. Count: 4.- (d = 25): (N = 16). Valid (y) values are coprime to 16. Count: 2.- (d = 50): (N = 4). Valid (y) values are coprime to 4. Count: 1.- (d = 100): (N = 1). No valid (y) as (x) must be positive. Count: 0. Summing All Counts:[40 + 20 + 20 + 8 + 4 + 4 + 2 + 1 + 0 = 99]Thus, the number of ordered pairs ((a, b)) is (boxed{99}).
🔑:To find the number of ordered pairs of positive integers (a, b) satisfying the equation[operatorname{gcd}(a, b) cdot a + b^2 = 10000,]we proceed as follows:1. Express using operatorname{gcd} transformation: - Let d = operatorname{gcd}(a, b). - Therefore, we can express a and b as a = d a' and b = d b', where operatorname{gcd}(a', b') = 1.2. Transform the original equation: - Substituting a = d a' and b = d b' into the equation gives: [ operatorname{gcd}(a, b) cdot a + b^2 = d cdot (d a') + (d b')^2 = d^2 a' + d^2 b'^2 = 10000. ] - This simplifies to: [ d^2 (a' + b'^2) = 10000. ] - We know 10000 = 100^2, so: [ d^2 (a' + b'^2) = 100^2. ] - Hence, a' + b'^2 = frac{10000}{d^2} = left(frac{100}{d}right)^2.3. Consider each divisor d of 100: - Recall that d divides 100; hence, we need to enumerate the divisors of 100 and solve for each one: [ d = 1, 2, 4, 5, 10, 20, 25, 50, 100. ]4. Solving for coprime integers ( a' ) and ( b' ): - For each divisor d, we need to find the number of solutions in coprime integers left(a', b'right) such that: [ a' + b'^2 = left(frac{100}{d}right)^2. ] - Note that every b' < frac{100}{d} coprime to d satisfies this equation.5. Counting coprime pairs: - The number of values b' (where 1 leq b' < frac{100}{d}) that are coprime to frac{100}{d} is given by Euler's totient function varphileft(frac{100}{d}right). - Summing these values for all d and subtracting the overcounted solution shows: [ sum_{d mid 100} varphileft(frac{100}{d}right) - 1. ] - For d = 100, we count the solution (0, 100), which is not valid in our scenario, thus subtract: [ 100 - 1 = 99. ]Thus, the total number of ordered pairs (a, b) is:[boxed{99}]
❓:Given the sequence ({a_n}) satisfying (a_1 = 1) and (a_n = n^2 left(1 + frac{1}{2^2} + frac{1}{3^2} + cdots + frac{1}{(n-1)^2}right)) for (n = 2, 3, cdots), prove:1. (frac{a_n + 1}{a_{n+1}} = frac{n^2}{(n+1)^2}) for (n = 2, 3, cdots).2. (left(1 + frac{1}{a_1}right)left(1 + frac{1}{a_2}right) cdots left(1 + frac{1}{a_n}right) < 4) for (n = 1, 2, cdots).
🧠:Okay, let's try to tackle these two problems. Both of them are about the sequence defined by a₁ = 1 and aₙ = n²(1 + 1/2² + ... + 1/(n-1)²) for n ≥ 2. The first problem is to prove that (aₙ + 1)/a_{n+1} = n²/(n+1)² for n ≥ 2. The second one is to show that the product (1 + 1/a₁)(1 + 1/a₂)...(1 + 1/aₙ) is less than 4 for any n. Let's start with the first problem.First, let's make sure I understand the sequence. For n=2, a₂ = 2²(1) = 4, since the sum inside is from 1 to 1, which is just 1. For n=3, a₃ = 3²(1 + 1/2²) = 9*(1 + 1/4) = 9*(5/4) = 45/4 = 11.25. Similarly, a₄ = 4²(1 + 1/2² + 1/3²) = 16*(1 + 1/4 + 1/9) = 16*(49/36) = 196/9 ≈ 21.78. Okay, that seems to be the pattern.Now, problem 1: Show that (aₙ + 1)/a_{n+1} = n²/(n+1)² for n ≥ 2. Let's try to compute a_{n+1} first. By definition, a_{n+1} = (n+1)²(1 + 1/2² + ... + 1/(n)^2). So the sum inside is up to 1/n². But aₙ is n² times the sum up to 1/(n-1)². So maybe we can relate the sum in a_{n+1} to the sum in aₙ plus 1/n².Let me write down:a_{n+1} = (n+1)² [ (1 + 1/2² + ... + 1/(n-1)²) + 1/n² ]But the sum inside aₙ is S_{n-1} = 1 + 1/2² + ... + 1/(n-1)². So aₙ = n² * S_{n-1}, which means S_{n-1} = aₙ / n². Therefore, a_{n+1} = (n+1)² [ S_{n-1} + 1/n² ] = (n+1)² [ aₙ / n² + 1/n² ] = (n+1)² (aₙ + 1)/n².Therefore, a_{n+1} = (n+1)² (aₙ + 1)/n². Let's rearrange this equation to solve for (aₙ + 1)/a_{n+1}:(aₙ + 1)/a_{n+1} = n²/(n+1)². Which is exactly what we needed to prove. So that's straightforward once you express a_{n+1} in terms of aₙ. So problem 1 is done. That was actually simpler than I initially thought. Just substitute the expression for a_{n+1} using the previous term and manipulate the equation.Now, moving on to problem 2: Prove that the product (1 + 1/a₁)(1 + 1/a₂)...(1 + 1/aₙ) < 4 for all n ≥ 1. Hmm. Let's compute the first few terms to get a sense.For n=1: The product is just (1 + 1/a₁) = 1 + 1/1 = 2 < 4.For n=2: (1 + 1/a₁)(1 + 1/a₂) = 2 * (1 + 1/4) = 2 * 5/4 = 10/4 = 2.5 < 4.For n=3: 2.5 * (1 + 1/a₃). a₃ is 45/4, so 1/a₃ = 4/45. So 1 + 1/a₃ = 49/45 ≈ 1.088. So the product becomes 2.5 * 49/45 ≈ 2.5 * 1.088 ≈ 2.72 < 4.Similarly, n=4: a₄ = 196/9, so 1 + 1/a₄ = 1 + 9/196 = 205/196 ≈ 1.045. Multiply previous product: 2.72 * 1.045 ≈ 2.84. Still way below 4. So it seems the product is increasing but maybe approaching a limit less than 4. So we need to show that this product is always less than 4.How can we approach this? Maybe use induction or find a telescoping product using the relation from problem 1. Since problem 1 gives a relation between aₙ and a_{n+1}, perhaps that can help in simplifying the product.Let me recall that in problem 1, we have (aₙ + 1)/a_{n+1} = n²/(n+1)². Let's rearrange this equation:1 + 1/aₙ = (aₙ + 1)/aₙ = [ (n²/(n+1)²) * a_{n+1} ] / aₙ.Wait, let's see. From problem 1: (aₙ + 1)/a_{n+1} = n²/(n+1)². Therefore, (aₙ + 1) = a_{n+1} * (n²)/(n+1)². Therefore, 1 + 1/aₙ = (a_{n+1}/aₙ) * (n²)/(n+1)².Hmm, but that might not directly help. Alternatively, maybe express 1 + 1/aₙ in terms of a_{n+1} and aₙ. Wait, 1 + 1/aₙ = (aₙ + 1)/aₙ = [ (n²/(n+1)²) * a_{n+1} ] / aₙ. So that's (n²/(n+1)²) * (a_{n+1}/aₙ). But I'm not sure if that helps.Alternatively, perhaps we can relate each term (1 + 1/aₖ) to a ratio involving a_{k+1} and a_k. Let me try that.From problem 1, we have:(aₖ + 1)/a_{k+1} = k²/(k+1)²Multiply both sides by a_{k+1}/(aₖ + 1):1 = [k²/(k+1)²] * a_{k+1}/(aₖ + 1)Wait, not sure. Alternatively, from (aₖ + 1)/a_{k+1} = k²/(k+1)², rearranged:(aₖ + 1) = a_{k+1} * (k²)/(k+1)²So 1 + 1/aₖ = (a_{k+1}/aₖ) * (k²)/(k+1)²Therefore, 1 + 1/aₖ = (k²/(k+1)²) * (a_{k+1}/aₖ)Therefore, the product from k=1 to n of (1 + 1/aₖ) is the product from k=1 to n of [ (k²/(k+1)²) * (a_{k+1}/aₖ) ].Let me write that out:Product_{k=1}^n (1 + 1/aₖ) = Product_{k=1}^n [ (k²/(k+1)²) * (a_{k+1}/aₖ) ]This product can be split into two separate products:= [ Product_{k=1}^n (k²/(k+1)²) ] * [ Product_{k=1}^n (a_{k+1}/aₖ) ]Let's compute each product separately.First product: Product_{k=1}^n (k²/(k+1)²) = [ (1²/2²) * (2²/3²) * ... * (n²/(n+1)²) ) ] = (1² * 2² * ... * n²)/(2² * 3² * ... * (n+1)²)) = ( (n! )² ) / ( ( (n+1)! )² / (1²) )) ) Wait, perhaps telescoping.Note that each numerator term cancels with the denominator of the previous term. For example, the numerator of the first term is 1², denominator is 2². Numerator of the second term is 2², denominator is 3². So when you multiply all terms, the 2² in the numerator cancels with the 2² in the denominator of the next term? Wait, no, actually, in denominators. Wait, let me think:Product_{k=1}^n (k/(k+1))² = [1/2 * 2/3 * 3/4 * ... * n/(n+1)]² = [1/(n+1)]². Because the product telescopes: 1/2 * 2/3 * ... * n/(n+1) = 1/(n+1). Therefore, squared, it's 1/(n+1)².So the first product is 1/(n+1)².Second product: Product_{k=1}^n (a_{k+1}/aₖ). This is a telescoping product as well. Each a_{k+1}/aₖ will multiply to a_{n+1}/a₁. Because:(a_{2}/a₁)*(a_{3}/a₂)*(a₄/a₃)*...*(a_{n+1}/aₙ) = a_{n+1}/a₁.Therefore, the second product is a_{n+1}/a₁. But a₁ = 1, so it's a_{n+1}/1 = a_{n+1}.Therefore, combining both products:Product_{k=1}^n (1 + 1/aₖ) = [1/(n+1)²] * a_{n+1}So, the product is equal to a_{n+1}/(n+1)². Therefore, we have:Product_{k=1}^n (1 + 1/aₖ) = a_{n+1}/(n+1)²But wait, the problem asks us to show that this product is less than 4. So we need to show that a_{n+1}/(n+1)² < 4.But from the definition of aₙ, a_{n+1} = (n+1)² [1 + 1/2² + ... + 1/n²]. Therefore, a_{n+1}/(n+1)² = 1 + 1/2² + ... + 1/n².Wait, so this means the product is equal to the sum 1 + 1/2² + ... + 1/n². Therefore, we need to show that 1 + 1/2² + ... + 1/n² < 4 for all n ≥ 1.But wait, the sum 1 + 1/4 + 1/9 + ... + 1/n² is a well-known convergent series that converges to π²/6 ≈ 1.6449. So even as n approaches infinity, the sum approaches approximately 1.6449, which is much less than 4. Therefore, for any finite n, the sum is less than π²/6, which is less than 4. Therefore, the product is equal to the partial sum of the Basel problem, which is always less than π²/6 < 4.Therefore, since 1 + 1/2² + ... + 1/n² < π²/6 < 4, we have that the product is less than 4. Hence, part 2 is proven.But let me verify this reasoning again. The key step is recognizing that the product telescopes to a_{n+1}/(n+1)^2, which is equal to the sum S_n = 1 + 1/2² + ... + 1/n². Then since S_n converges to π²/6 ≈ 1.6449, which is indeed less than 4. Therefore, for all n, S_n < π²/6 < 4.But perhaps we need a more elementary argument that doesn't rely on knowing the exact limit of the Basel problem. Because maybe the problem expects us to bound the sum without referencing π²/6.Alternatively, maybe use induction. Let's see.Base case n=1: product is 2 < 4, true. Assume that for some k ≥1, the product up to k terms is less than 4. Then for k+1, the product is previous product times (1 + 1/a_{k+1}). Need to show it remains less than 4. But this might not be straightforward, as we need to bound how much the product increases each time.Alternatively, note that the sum 1 + 1/2² + ... + 1/n² is less than 1 + 1/(1*2) + 1/(2*3) + ... + 1/(n(n-1)) for n ≥2. Wait, but 1/(k(k-1)) = 1/(k-1) - 1/k. So that sum telescopes to 2 - 1/n. Wait, but 1 + 1/(1*2) + 1/(2*3) + ... +1/((n-1)n) = 1 + (1 - 1/2) + (1/2 - 1/3) + ... + (1/(n-1) -1/n)) = 1 +1 -1/n = 2 -1/n < 2.But since 1/k² < 1/(k(k-1)) for k ≥2, the sum 1 +1/2² +1/3² + ... +1/n² < 1 + [1/(1*2) +1/(2*3)+...+1/((n-1)n)] =1 + (1 -1/n) = 2 -1/n <2 <4. Wait, but if that's true, then the sum is less than 2, so the product would be less than 2, which is even better. But this contradicts the Basel problem's limit of π²/6≈1.6449. So there's an inconsistency here.Wait, let's see. The comparison 1/k² < 1/(k(k-1)) is valid for k ≥2. Because 1/k² = 1/(k*k) and 1/(k(k-1)) = 1/(k² -k). Since k² -k <k², then 1/(k² -k) >1/k². So actually, 1/k² <1/(k(k-1)). Therefore, the sum 1 +1/2² + ... +1/n² <1 + [1/(1*2) +1/(2*3) + ... +1/((n-1)n)] =1 + (1 -1/2 +1/2 -1/3 + ... +1/(n-1) -1/n) )=1 + (1 -1/n) =2 -1/n <2. Therefore, the sum S_n <2 -1/n <2. Therefore, the product is S_n <2 <4. So even a stronger statement holds. But in reality, the Basel problem sum is about 1.64, so this method gives an upper bound of 2, which is still less than 4. Therefore, the product is less than 2, which is less than 4. Therefore, the inequality holds.But the problem states to prove it's less than 4. So maybe the problem is easier than expected because we can use a telescoping sum to bound S_n <2. However, let's verify with n=1: sum is 1 <2. n=2:1 +1/4=1.25<2. n=3:1 +1/4 +1/9≈1.361<2. And so on. Yes, the sum is approaching π²/6≈1.64, so it's always less than 2. Hence, the product is less than 2, which is way less than 4.But maybe the problem is misstated? Or perhaps I made a mistake in the telescoping.Wait, going back: I said that the product equals a_{n+1}/(n+1)², which is equal to the sum S_n =1 +1/2² +... +1/n². But according to the definition, a_{n+1} = (n+1)² * S_n. Therefore, a_{n+1}/(n+1)^2 = S_n. So yes, Product_{k=1}^n (1 +1/a_k) = S_n. Therefore, since S_n is the partial sum of the convergent series, which converges to π²/6≈1.6449, and each partial sum is less than this limit. Hence, S_n < π²/6 <4. So the product is indeed less than 4. But actually, even better, it's less than 2.Wait, but according to the telescoping sum comparison, we have S_n <2 -1/n <2. So S_n <2. Therefore, the product is less than 2. Hence, the problem's upper bound of 4 is not tight. But since the question asks to prove it's less than 4, then our conclusion still holds. However, the stronger bound of 2 is also true. Maybe the problem just wants 4 for simplicity.Alternatively, perhaps my earlier step where I split the product into two parts and found that it equals S_n is correct, but I should double-check.Let me recapitulate:We have for each k ≥1:1 +1/a_k = (a_k +1)/a_k = [ (k²/(k+1)²) * a_{k+1} ] /a_k. This comes from problem 1, which was for n≥2. Wait, problem 1 states the identity for n≥2. But here, in the product, we have terms starting from k=1. However, for k=1, we need to check if the relation (a₁ +1)/a₂ =1²/2². Let's verify that.Given a₁=1, a₂=4. Then (a₁ +1)/a₂=(1 +1)/4=2/4=1/2. And 1²/2²=1/4. Wait, 1/2 ≠1/4. So the identity in problem 1 is only valid for n≥2, but in the product, the term for k=1 is (1 +1/a₁) which is 2. So maybe the telescoping product approach works only for k≥2. Therefore, my previous reasoning might have a mistake here.Ah, here's the error. The identity from problem 1 is valid for n≥2, which corresponds to k≥2 in the product. However, the term k=1 in the product is (1 +1/a₁)=2, which is not covered by problem 1. Therefore, my earlier approach that split the product into terms from k=1 to n using the identity is incorrect for k=1. Therefore, the mistake is that the identity (aₖ +1)/a_{k+1}=k²/(k+1)² is only valid for k≥2. Therefore, when we consider the product from k=1 to n, the first term (k=1) doesn't satisfy the relation used. Therefore, my previous derivation is flawed.This is a critical mistake. Let's correct this.So, let's adjust the approach. For k≥2, we have the identity (aₖ +1)/a_{k+1}=k²/(k+1)². Therefore, for k≥2, 1 +1/aₖ= (k²/(k+1)²)*(a_{k+1}/aₖ). However, for k=1, we need to treat it separately.Therefore, the product from k=1 to n of (1 +1/a_k) = (1 +1/a₁) * Product_{k=2}^n (1 +1/a_k).We already know that (1 +1/a₁)=2. Then, for k≥2, each term (1 +1/a_k) can be expressed as (k²/(k+1)²)*(a_{k+1}/a_k). Therefore, the product from k=2 to n is Product_{k=2}^n [ (k²/(k+1)²)*(a_{k+1}/a_k) ].This can be split into two products:Product_{k=2}^n (k²/(k+1)²) * Product_{k=2}^n (a_{k+1}/a_k).First product: Product_{k=2}^n (k²/(k+1)²) = [2²/3² *3²/4²*...*n²/(n+1)²] = (2²)/(n+1)². Because it telescopes: numerator of each term cancels denominator of the previous term. So 2² remains in the numerator, and (n+1)² in the denominator.Second product: Product_{k=2}^n (a_{k+1}/a_k) = a_{n+1}/a₂. Because telescoping: each a_{k+1}/a_k multiplies to a_{n+1}/a₂.Therefore, the product from k=2 to n is [ (2²)/(n+1)² ] * [ a_{n+1}/a₂ ].Therefore, the total product from k=1 to n is:2 * [ (4)/(n+1)² ] * [ a_{n+1}/4 ] = 2 * [ a_{n+1}/(n+1)² ] * (4/4) )? Wait, let me compute step by step.Wait, a₂=4. Therefore, a_{n+1}/a₂ = a_{n+1}/4.So the product from k=2 to n is [ (2² *3² *...*n²)/(3²*4²*...*(n+1)²) ) ] * [a_{n+1}/4].Wait, no, the first product was [2²/(n+1)²], as explained earlier. Because when you multiply (2²/3²)*(3²/4²)*...*(n²/(n+1)²) =2²/(n+1)². So first product is 2²/(n+1)², second product is a_{n+1}/a₂ =a_{n+1}/4.Thus, the product from k=2 to n is (4/(n+1)²)*(a_{n+1}/4) )= a_{n+1}/(n+1)².Therefore, the total product from k=1 to n is:(1 +1/a₁) * [a_{n+1}/(n+1)²] =2 * [a_{n+1}/(n+1)²].But a_{n+1}/(n+1)² is equal to S_n =1 +1/2² + ... +1/n². Therefore, the total product is 2*S_n.Wait, earlier I thought it was equal to S_n, but that was incorrect because I included k=1 in the product which required separate handling. Now, with correct handling, the product is 2*S_n.Therefore, the product (1 +1/a₁)...(1 +1/aₙ) =2*(1 +1/4 +1/9 +...+1/n²).Therefore, to show that this product is less than 4, we need to show that 2*S_n <4, i.e., S_n <2. Which we already established by comparing to the telescoping series. Since S_n <2 -1/n <2, then 2*S_n <4 -2/n <4. Therefore, the product is less than 4 -2/n <4. Therefore, the inequality holds.Thus, even though the initial approach had a mistake regarding the k=1 term, correcting it leads to the product being 2*S_n, and since S_n <2, then 2*S_n <4. Hence, part 2 is proven.To summarize:1. For the first part, express a_{n+1} in terms of aₙ and use the given recursive formula to establish the required identity.2. For the second part, relate each term (1 +1/aₖ) to a_{k+1} and use telescoping products to express the entire product in terms of a_{n+1} and n. Correctly accounting for the first term leads to the product being twice the partial sum of the reciprocals of squares, which is bounded above by 2, leading to the product being less than 4.Therefore, both statements are proven.
✅:Solution:1. Proving (frac{a_n + 1}{a_{n+1}} = frac{n^2}{(n+1)^2}) for (n geq 2):Given the sequence (a_n = n^2 left(1 + frac{1}{2^2} + cdots + frac{1}{(n-1)^2}right)) for (n geq 2), consider (a_{n+1}):[a_{n+1} = (n+1)^2 left(1 + frac{1}{2^2} + cdots + frac{1}{(n-1)^2} + frac{1}{n^2}right).]Let (S_{n-1} = 1 + frac{1}{2^2} + cdots + frac{1}{(n-1)^2}). Then (a_n = n^2 S_{n-1}), so:[a_{n+1} = (n+1)^2 left(S_{n-1} + frac{1}{n^2}right) = (n+1)^2 left(frac{a_n}{n^2} + frac{1}{n^2}right) = frac{(n+1)^2}{n^2} (a_n + 1).]Rearranging gives:[frac{a_n + 1}{a_{n+1}} = frac{n^2}{(n+1)^2}.]2. Proving (left(1 + frac{1}{a_1}right)cdotsleft(1 + frac{1}{a_n}right) < 4) for all (n geq 1):First, note (a_{n+1}/(n+1)^2 = 1 + frac{1}{2^2} + cdots + frac{1}{n^2} = S_n). Using the result from part 1 for (k geq 2):[1 + frac{1}{a_k} = frac{k^2}{(k+1)^2} cdot frac{a_{k+1}}{a_k}.]The product from (k=2) to (n) telescopes:[prod_{k=2}^n left(1 + frac{1}{a_k}right) = prod_{k=2}^n left(frac{k^2}{(k+1)^2} cdot frac{a_{k+1}}{a_k}right) = frac{4}{(n+1)^2} cdot frac{a_{n+1}}{4} = frac{a_{n+1}}{(n+1)^2} = S_n.]Including the (k=1) term ((1 + 1/a_1) = 2), the total product becomes:[2 cdot S_n = 2 left(1 + frac{1}{2^2} + cdots + frac{1}{n^2}right).]Since the series (sum_{k=1}^infty frac{1}{k^2}) converges to (frac{pi^2}{6} approx 1.6449), we have (S_n < frac{pi^2}{6} < 2). Thus:[2 cdot S_n < 2 cdot 2 = 4.]Final Answer:1. (boxed{frac{n^2}{(n+1)^2}})2. (boxed{4})
🔑:We are given a sequence {a_n} such that:[ a_1 = 1 ][ a_n = n^2 left( 1 + frac{1}{2^2} + frac{1}{3^2} + cdots + frac{1}{(n-1)^2} right) text{ for } n = 2, 3, ldots ]We need to prove two statements.# (1) Prove that (frac{a_n + 1}{a_{n+1}} = frac{n^2}{(n+1)^2})1. Starting from the definition of (a_n):[ a_n = n^2 left( 1 + frac{1}{2^2} + frac{1}{3^2} + cdots + frac{1}{(n-1)^2} right) ]2. Next, consider (a_{n+1}):[ a_{n+1} = (n+1)^2 left( 1 + frac{1}{2^2} + frac{1}{3^2} + cdots + frac{1}{n^2} right) ]3. Express (frac{a_{n+1}}{(n+1)^2}):[frac{a_{n+1}}{(n+1)^2} = 1 + frac{1}{2^2} + cdots + frac{1}{n^2}]4. Express (frac{a_n}{n^2}) in a different form:[frac{a_n}{n^2} = 1 + frac{1}{2^2} + cdots + frac{1}{(n-1)^2}]5. Given these two relationships, we can rewrite (frac{a_{n+1}}{(n+1)^2}) by adding (frac{1}{n^2}) to (frac{a_n}{n^2}):[frac{a_{n+1}}{(n+1)^2} = frac{a_n}{n^2} + frac{1}{n^2}]6. Thus, we can further express:[frac{a_{n+1}}{(n+1)^2} = frac{a_n + 1}{n^2}]7. Rearrange to find:[a_{n+1} = frac{(n+1)^2}{n^2} (a_n + 1)]8. Finally, verifying the required identity:[frac{a_n + 1}{a_{n+1}} = frac{n^2}{(n+1)^2}]This completes the proof for the first statement.[boxed{frac{a_n + 1}{a_{n+1}} = frac{n^2}{(n+1)^2} text{ for } n = 2, 3, ldots}]# (2) Prove that (left(1 + frac{1}{a_1}right)left(1 + frac{1}{a_2}right) cdots left(1 + frac{1}{a_n}right) < 4 text{ for } n = 1, 2, ldots)1. For (n=1):[1 + frac{1}{a_1} = 1 + frac{1}{1} = 2 < 4]2. For (n geq 2), using previously proven result (frac{a_n + 1}{a_{n+1}} = frac{n^2}{(n+1)^2}):[left(1 + frac{1}{a_1}right)left(1 + frac{1}{a_2}right) cdots left(1 + frac{1}{a_n}right) = frac{a_1 + 1}{a_1} frac{a_2 + 1}{a_2} cdots frac{a_n + 1}{a_n}]3. Using the identity, this can be expressed as:[= frac{a_1 + 1}{a_1 a_2 cdots a_n} cdot frac{a_2 + 1}{a_3} cdots frac{a_n + 1}{a_{n+1}}]4. Notice that each subsequent term simplifies:[= frac{2}{1} cdot left(frac{2^2}{3^2} cdot frac{3^2}{4^2} cdots frac{n^2}{(n+1)^2}right) a_{n+1}]5. This is equivalent to:[= 2 cdot frac{a_{n+1}}{(n+1)^2}]6. Substitute (a_{n+1}):[= 2 left(1 + frac{1}{2^2} + frac{1}{3^2} + cdots + frac{1}{n^2}right)]7. Recognize the sum of series approximation:[< 2 left( 1 + frac{1}{1 times 2} + frac{1}{2 times 3} + cdots + frac{1}{(n-1) n} right)]8. This is a telescoping series:[= 2 left( 1 + left(1 - frac{1}{2}right) + left(frac{1}{2} - frac{1}{3}right) + cdots + left(frac{1}{n-1} - frac{1}{n}right) right)]9. The series simplifies to:[= 2 left(2 - frac{1}{n}right) < 4]Thus, we have proved that for all (n = 1, 2, ldots):[left(1 + frac{1}{a_1}right)left(1 + frac{1}{a_2}right) cdots left(1 + frac{1}{a_n}right) < 4][boxed{left(1 + frac{1}{a_1}right)left(1 + frac{1}{a_2}right) cdots left(1 + frac{1}{a_n}right) < 4 text{ for } n=1, 2, ldots}]
❓:Positive numbers (a, b), and (c) are such that (a cdot b cdot c = 1). Prove the inequalityfrac{1}{1+a+b}+frac{1}{1+b+c}+frac{1}{1+c+a} leq 1
🧠:Okay, so I need to prove that for positive numbers a, b, c with abc = 1, the sum of 1/(1+a+b) + 1/(1+b+c) + 1/(1+c+a) is less than or equal to 1. Hmm, let's start by understanding the problem.First, the condition is that the product abc = 1. That makes me think maybe using substitution based on this condition could help. Sometimes with symmetric inequalities involving products equal to 1, substituting variables as ratios or using logarithms might be useful, but I'm not sure yet.Alternatively, maybe I can use the AM-GM inequality here since we have positive numbers and products. Let me recall that AM-GM states that for non-negative numbers, the arithmetic mean is greater than or equal to the geometric mean. So for two numbers x and y, (x + y)/2 ≥ √(xy). Maybe this can be applied to the denominators somehow.Looking at each term in the left-hand side (LHS) of the inequality: 1/(1 + a + b). Since the denominators are sums, maybe there's a way to bound each term from above. If I can find a lower bound for each denominator, then the entire fraction would have an upper bound.Wait, but how to relate this to the condition abc = 1? Let me see. If abc = 1, perhaps we can express one variable in terms of the others. For example, c = 1/(ab). Then substitute this into the inequality to reduce the number of variables. Let's try that.Substituting c = 1/(ab), the inequality becomes:1/(1 + a + b) + 1/(1 + b + 1/(ab)) + 1/(1 + 1/(ab) + a) ≤ 1.Hmm, this seems complicated. Maybe there's a better substitution. Alternatively, maybe homogenizing the inequality. Since abc = 1, we can scale the variables if needed, but they are already positive.Wait, homogenization is a technique where we make all terms have the same degree. Let's check the degrees. The denominators in the fractions are 1 + a + b, which is degree 1 if we consider a, b, c as degree 1 variables. But since abc = 1, which is degree 3, maybe homogenizing by multiplying numerator and denominator by some expression.Alternatively, perhaps using the substitution a = x/y, b = y/z, c = z/x. Then abc = (x/y)*(y/z)*(z/x) = 1, which satisfies the condition. Let's see if this substitution helps.But maybe that's overcomplicating. Let me think again. Another approach is to use Cauchy-Schwarz inequality. Let me recall that Cauchy-Schwarz can be used in the form (sum (1/(1 + a + b))) * (sum (1 + a + b)) ≥ (1 + 1 + 1)^2 = 9. But then sum (1/(1 + a + b)) ≥ 9 / sum(1 + a + b). However, this gives a lower bound, but we need an upper bound. So maybe this is not directly helpful.Alternatively, maybe using the Titu's lemma, which is a form of Cauchy-Schwarz: sum (a_i^2 / b_i) ≥ (sum a_i)^2 / sum b_i. But again, not sure how to apply here.Wait, let's consider another angle. Maybe we can compare each term to something. Let's suppose that since abc = 1, and all variables are positive, perhaps using the substitution a = x^2, b = y^2, c = z^2 with xyz = 1. Not sure if that helps.Alternatively, maybe let’s consider the symmetry of the inequality. The inequality is cyclic in a, b, c. So perhaps assuming WLOG that a ≥ b ≥ c or something. But since the condition is symmetric, maybe not helpful.Wait, another thought: Let’s make the substitution a = x/k, b = y/k, c = z/k such that xyz = k^3. But since abc = 1, then (x/k)(y/k)(z/k) = xyz/k^3 = 1, so xyz = k^3. Hmm, maybe this substitution is not helpful.Alternatively, maybe setting a = x/y, b = y/z, c = z/x as before. Then substituting into the inequality:1/(1 + x/y + y/z) + 1/(1 + y/z + z/x) + 1/(1 + z/x + x/y) ≤ 1.But this seems messy. Let me check if there's an identity or known inequality related to this form.Alternatively, maybe adding up all the denominators. Let’s denote S = [1 + a + b] + [1 + b + c] + [1 + c + a] = 3 + 2(a + b + c). But not sure how that helps.Alternatively, trying specific values to check if the inequality holds. For example, take a = b = c = 1, since 1*1*1 = 1. Then the LHS is 1/(1 + 1 + 1) + 1/(1 + 1 + 1) + 1/(1 + 1 + 1) = 3*(1/3) = 1. So equality holds here. That's a good check.What if we take a = 2, b = 2, then c = 1/(2*2) = 1/4. Then compute LHS:1/(1 + 2 + 2) = 1/5,1/(1 + 2 + 1/4) = 1/(1 + 2 + 0.25) = 1/3.25 ≈ 0.3077,1/(1 + 1/4 + 2) = 1/(3.25) ≈ 0.3077.Sum ≈ 0.2 + 0.3077 + 0.3077 ≈ 0.8154 ≤ 1. So it holds here.Another test case: a = 3, b = 1, so c = 1/(3*1) = 1/3.Compute each term:1/(1 + 3 + 1) = 1/5 = 0.2,1/(1 + 1 + 1/3) = 1/(2 + 1/3) = 1/(7/3) ≈ 0.4286,1/(1 + 1/3 + 3) = 1/(4 + 1/3) = 1/(13/3) ≈ 0.2308.Total ≈ 0.2 + 0.4286 + 0.2308 ≈ 0.8594 ≤ 1. Still holds.Another test case: a = 1/2, b = 1/2, so c = 1/( (1/2)(1/2) ) = 4.Compute terms:1/(1 + 1/2 + 1/2) = 1/2,1/(1 + 1/2 + 4) = 1/(5.5) ≈ 0.1818,1/(1 + 4 + 1/2) = 1/5.5 ≈ 0.1818.Sum ≈ 0.5 + 0.1818 + 0.1818 ≈ 0.8636 ≤ 1. Still holds.So far, the inequality seems to hold in test cases. Now, how to approach the proof.Let me consider the denominators. Each denominator is 1 + a + b, 1 + b + c, 1 + c + a. Since abc = 1, perhaps using the substitution c = 1/(ab) to express everything in terms of a and b. But that might complicate things.Alternatively, maybe using the AM-GM inequality on the denominator. For example, 1 + a + b ≥ 3*(1*a*b)^{1/3} by AM-GM. But 1*a*b = ab, so (ab)^{1/3}. Then 1 + a + b ≥ 3*(ab)^{1/3}. Then 1/(1 + a + b) ≤ 1/(3*(ab)^{1/3}).But since abc = 1, c = 1/(ab), so (ab)^{1/3} = (ab)^{1/3} = (ab)^{1/3} and c = 1/(ab), so maybe there is a relation here.Wait, (ab)^{1/3} = (ab)^{1/3} = (ab)^{1/3}, but since c = 1/(ab), then (ab) = 1/c. Therefore, (ab)^{1/3} = (1/c)^{1/3} = c^{-1/3}. So 1/(3*(ab)^{1/3}) = c^{1/3}/3.Therefore, 1/(1 + a + b) ≤ c^{1/3}/3. Similarly, the other terms:1/(1 + b + c) ≤ a^{1/3}/3,1/(1 + c + a) ≤ b^{1/3}/3.Adding them up: (c^{1/3} + a^{1/3} + b^{1/3}) / 3. But by AM-GM, (a^{1/3} + b^{1/3} + c^{1/3}) / 3 ≥ (a^{1/3}b^{1/3}c^{1/3})^{1/3} = ( (abc)^{1/3} )^{1/3} = (1^{1/3})^{1/3} = 1^{1/9} = 1. Wait, no, AM-GM says that the arithmetic mean is greater than or equal to the geometric mean. So (a^{1/3} + b^{1/3} + c^{1/3}) / 3 ≥ (a^{1/3}b^{1/3}c^{1/3})^{1/3} )? Wait, no. Let me check.The geometric mean of a^{1/3}, b^{1/3}, c^{1/3} is (a^{1/3}b^{1/3}c^{1/3})^{1/3} = ( (abc)^{1/3} )^{1/3} = (1^{1/3})^{1/3} = 1^{1/9} = 1. Wait, that seems off. Wait, let's compute geometric mean correctly. For three variables x, y, z, geometric mean is (xyz)^{1/3}. So if x = a^{1/3}, y = b^{1/3}, z = c^{1/3}, then geometric mean is (a^{1/3}b^{1/3}c^{1/3})^{1/3} = ( (abc)^{1/3} )^{1/3} = abc^{1/9} = 1^{1/9} = 1. But since abc = 1. Therefore, the arithmetic mean is (a^{1/3} + b^{1/3} + c^{1/3}) / 3 ≥ 1. Therefore, a^{1/3} + b^{1/3} + c^{1/3} ≥ 3. Therefore, (c^{1/3} + a^{1/3} + b^{1/3}) / 3 ≥ 1. Therefore, sum of the three terms would be ≤ (something ≥1), but we need sum ≤1. Therefore, this approach might not work, as it gives an upper bound in the wrong direction.Hmm, maybe that path is not helpful. Let me think again.Another idea: Let's make the substitution x = a, y = b, z = c, with xyz = 1. Then we need to show that:1/(1 + x + y) + 1/(1 + y + z) + 1/(1 + z + x) ≤ 1.Alternatively, perhaps we can use the Cauchy-Schwarz inequality in a different way. Let me consider that:Sum [1/(1 + a + b)] ≤ 1.Alternatively, maybe consider that each term can be written as 1/(1 + a + b) = 1/(1 + a + b) * (c/c) = c/(c + ac + bc). Since abc = 1, then ac = 1/b, bc = 1/a. Therefore, denominator becomes c + 1/b + 1/a. Hmm, so 1/(1 + a + b) = c/(c + 1/b + 1/a). Not sure if this helps.Wait, let's write it down:1/(1 + a + b) = c/(c + (1 + a + b)c) = c/(c + c + ac + bc). But since abc = 1, ac = 1/b and bc = 1/a. Therefore, denominator becomes c + c + 1/b + 1/a = 2c + 1/a + 1/b. Hmm, not sure.Alternatively, maybe using the substitution in another way. Let's note that since abc = 1, we can set a = x/y, b = y/z, c = z/x for some positive x, y, z. Then substituting into the inequality:1/(1 + x/y + y/z) + 1/(1 + y/z + z/x) + 1/(1 + z/x + x/y) ≤ 1.But this seems complex. Let me see if multiplying numerator and denominator by something can help. For example, for the first term, multiply numerator and denominator by yz:yz/(yz + xz + y^2) ≤ ?Not sure. Maybe not helpful.Alternatively, let's consider the following approach: Let’s denote that since abc = 1, we can apply the substitution a = x/y, b = y/z, c = z/x as before, but maybe it's better to consider logarithms. Let’s set a = e^x, b = e^y, c = e^z, with x + y + z = 0. But this might complicate things further.Wait, another idea: Use the substitution a = x^2, b = y^2, c = z^2, so that x^2 y^2 z^2 = 1, hence xyz = 1 or -1, but since they are positive, xyz = 1. Then, the inequality becomes:1/(1 + x^2 + y^2) + 1/(1 + y^2 + z^2) + 1/(1 + z^2 + x^2) ≤ 1.Not sure if this helps. Maybe using Cauchy-Schwarz here:For example, (1 + x^2 + y^2)(1 + 1 + 1) ≥ (1 + x + y)^2. But this gives 1/(1 + x^2 + y^2) ≤ 3/(1 + x + y)^2. But then sum of such terms would be ≤ 3 sum 1/(1 + x + y)^2. Not sure if helpful.Alternatively, maybe consider the function f(a, b, c) = sum 1/(1 + a + b) and try to maximize it under the constraint abc = 1. Using Lagrange multipliers. Let me try that.Set up the Lagrangian: L = [1/(1 + a + b) + 1/(1 + b + c) + 1/(1 + c + a)] - λ(abc - 1).Take partial derivatives with respect to a, b, c, set to zero.Compute derivative with respect to a:dL/da = -1/(1 + a + b)^2 - 1/(1 + c + a)^2 - λ bc = 0.Similarly, derivative with respect to b:dL/db = -1/(1 + a + b)^2 -1/(1 + b + c)^2 - λ ac = 0.Derivative with respect to c:dL/dc = -1/(1 + b + c)^2 -1/(1 + c + a)^2 - λ ab = 0.And the constraint abc = 1.This system of equations seems symmetric. Perhaps the maximum occurs when a = b = c. Let's check if this is possible.If a = b = c, then since abc = 1, a = 1. Then the sum becomes 3/(1 + 1 + 1) = 1, which matches equality. So maybe the maximum is achieved at a = b = c = 1, and thus the inequality holds. However, to confirm that this is indeed the maximum, we need to check if the function f(a, b, c) attains its maximum when a = b = c = 1. But how?Alternatively, maybe using the method of substitution. Let’s set a = x, b = y, c = 1/(xy). Then f(a, b, c) becomes:1/(1 + x + y) + 1/(1 + y + 1/(xy)) + 1/(1 + 1/(xy) + x).Now, this is a function of two variables x and y. To find its maximum, set partial derivatives to zero. But this might be complicated.Alternatively, maybe consider that the function is symmetric and convex or concave, so the maximum occurs at the symmetric point. But without rigorous proof, this is just an intuition.Alternatively, maybe using the substitution u = a + b + c, v = ab + bc + ca, w = abc = 1. But not sure.Wait, another thought: Let’s apply the Cauchy-Schwarz inequality in the following way. For each term 1/(1 + a + b), note that 1 + a + b ≥ 1 + 2√(ab) by AM-GM on a and b. Then 1/(1 + a + b) ≤ 1/(1 + 2√(ab)).But since abc = 1, √(ab) = √(ab) = √(1/c). So 1/(1 + 2√(ab)) = 1/(1 + 2/√c). Similarly for other terms.Therefore, the sum becomes:Sum 1/(1 + 2/√c) ≤ 1.But this seems different. Let me check with a = b = c =1: each term is 1/(1 + 2/1) = 1/3, sum is 1, which matches. For a=2, b=2, c=1/4: each term becomes 1/(1 + 2/√(1/4)) = 1/(1 + 2/(1/2)) = 1/(1 + 4) = 1/5. But then sum would be 3*(1/5) = 3/5 < 1. But original sum was ≈0.8154. So this approach gives a weaker upper bound, but the actual sum is higher. Therefore, this approach might not be helpful since the upper bound is not tight enough.Alternatively, maybe another inequality. Let’s consider the following identity: For positive real numbers, 1/(1 + a + b) ≤ (1 + c)/9*(1 + 1 + 1 + a + b + c). Wait, not sure.Alternatively, let’s use the substitution t = a + b + c. Then, maybe relate t to the sum. But not sure.Wait, here's an idea: Let's use the Cauchy-Schwarz inequality on the sum.Let’s consider that:[sum 1/(1 + a + b)] * [sum (1 + a + b)] ≥ (1 + 1 + 1)^2 = 9.Therefore, sum 1/(1 + a + b) ≥ 9 / [sum (1 + a + b)].But sum (1 + a + b) = 3 + 2(a + b + c). Therefore,sum 1/(1 + a + b) ≥ 9 / [3 + 2(a + b + c)].But we need to show that sum ≤ 1. However, this gives a lower bound. Not helpful for upper bound.Alternatively, reverse Cauchy-Schwarz? Not sure.Alternatively, let's think of each term 1/(1 + a + b). Maybe compare it to some expression involving c.Since abc = 1, c = 1/(ab). Maybe express 1 + a + b in terms of c.1 + a + b = 1 + a + b.But since c = 1/(ab), perhaps we can write ab = 1/c. So 1 + a + b ≥ 1 + 2√(ab) = 1 + 2/√c. Wait, that's the same as before.Alternatively, maybe use Hölder's inequality. Hölder's might be more effective here.Recall that Hölder's inequality states that (sum a_i b_i c_i) ≤ (sum a_i^p)^{1/p} (sum b_i^q)^{1/q} (sum c_i^r)^{1/r}} where 1/p + 1/q + 1/r = 1.But I need to think how to apply Hölder here.Alternatively, consider the following approach:Let’s denote the denominators as follows:D1 = 1 + a + b,D2 = 1 + b + c,D3 = 1 + c + a.We need to show that 1/D1 + 1/D2 + 1/D3 ≤ 1.Multiply both sides by D1 D2 D3. Then the inequality becomes:D2 D3 + D1 D3 + D1 D2 ≤ D1 D2 D3.Which can be rewritten as:D1 D2 D3 - D2 D3 - D1 D3 - D1 D2 ≥ 0.Factorizing:D1 D2 D3 - D2 D3 - D1 D3 - D1 D2 = D3(D1 D2 - D2 - D1) - D1 D2.Hmm, this seems complicated. Let me compute each term.First, expand D1 D2 D3:(1 + a + b)(1 + b + c)(1 + c + a). This expansion would be very tedious. Maybe not helpful.Alternatively, maybe consider that D1 D2 D3 - D2 D3 - D1 D3 - D1 D2 = D3(D1 D2 - D1 - D2) - D1 D2. Still not helpful.Alternatively, let's try specific substitutions. Let’s set x = a + 1, y = b + 1, z = c + 1. But since abc =1, not sure.Wait, another idea: Let's use the fact that for positive numbers, 1/(1 + a + b) ≤ (1 + c)/9*(some expression). Wait, not sure.Alternatively, let's consider the following identity:1/(1 + a + b) = 1/(1 + a + b) * c/c = c/(c + ac + bc).But since abc =1, ac = 1/b and bc =1/a. So denominator becomes c + 1/b + 1/a.Therefore, 1/(1 + a + b) = c/(c + 1/a + 1/b).Similarly for the other terms:1/(1 + b + c) = a/(a + 1/b + 1/c),1/(1 + c + a) = b/(b + 1/c + 1/a).So the sum becomes:c/(c + 1/a + 1/b) + a/(a + 1/b + 1/c) + b/(b + 1/c + 1/a).Hmm, does this form help? Let me see.Let’s denote 1/a = x, 1/b = y, 1/c = z. Then since abc =1, xyz =1. Then the sum becomes:(1/z)/(1/z + y + x) + (1/x)/(1/x + y + z) + (1/y)/(1/y + z + x).Simplify each term:First term: (1/z)/(1/z + x + y) = 1/(1 + z(x + y)).Similarly, second term: 1/(1 + x(y + z)),Third term: 1/(1 + y(z + x)).So the sum is:1/(1 + z(x + y)) + 1/(1 + x(y + z)) + 1/(1 + y(z + x)) ≤ 1.With xyz =1.Hmm, this seems similar to the original inequality, but with variables x, y, z. Not sure if this helps.Alternatively, since xyz =1, maybe set x = p/q, y = q/r, z = r/p. Then xyz = (p/q)(q/r)(r/p) =1. Then substituting:1/(1 + (r/p)(p/q + q/r)) + ... Not sure.Alternatively, let's consider homogenization. Since abc =1, let's make all terms in the inequality have the same degree.Original inequality:1/(1 + a + b) + 1/(1 + b + c) + 1/(1 + c + a) ≤1.Multiply numerator and denominator of each term by c, a, b respectively:c/(c + ac + bc) + a/(a + ab + ac) + b/(b + bc + ab) ≤1.But since abc =1, we have ac =1/b and bc =1/a, so:c/(c + 1/b + 1/a) + a/(a + ab + 1/b) + b/(b + 1/a + ab).But ab =1/c, so substituting ab =1/c:First term: c/(c + 1/b + 1/a) = c/(c + c + c) = c/(3c) =1/3. Wait, hold on:Wait, if ab =1/c, then 1/a = bc and 1/b = ac. Therefore, 1/a +1/b = bc + ac = c(a + b).Therefore, first term denominator: c + 1/b + 1/a = c + ac + bc = c(1 + a + b). Wait, but this is just c*(1 + a + b). Therefore, first term is c/(c*(1 + a + b)) =1/(1 + a + b). Which brings us back. So this substitution isn't simplifying.Wait, maybe another angle. Let’s assume that a, b, c are all equal to 1, which gives equality. Maybe we can use the method of Lagrange multipliers to show that the maximum of the left-hand side under the constraint abc=1 occurs at a=b=c=1.But this requires calculus and partial derivatives, which might be involved, but let's try.Define the function f(a,b,c) = 1/(1 + a + b) + 1/(1 + b + c) + 1/(1 + c + a) and the constraint g(a,b,c) = abc -1 =0.The method of Lagrange multipliers tells us that at the extremum, the gradient of f is proportional to the gradient of g.Compute the partial derivatives of f:∂f/∂a = -1/(1 + a + b)^2 -1/(1 + c + a)^2,∂f/∂b = -1/(1 + a + b)^2 -1/(1 + b + c)^2,∂f/∂c = -1/(1 + b + c)^2 -1/(1 + c + a)^2.Gradient of g: (bc, ac, ab).According to Lagrange multipliers, there exists λ such that:∂f/∂a = λ bc,∂f/∂b = λ ac,∂f/∂c = λ ab.Therefore:- [1/(1 + a + b)^2 + 1/(1 + c + a)^2 ] = λ bc,- [1/(1 + a + b)^2 + 1/(1 + b + c)^2 ] = λ ac,- [1/(1 + b + c)^2 + 1/(1 + c + a)^2 ] = λ ab.Assuming symmetry, suppose that a = b = c. Since abc =1, each is 1. Check if this satisfies the equations.Left-hand side of first equation: - [1/(1 +1 +1)^2 +1/(1 +1 +1)^2 ] = - [2/(9)] = -2/9.Right-hand side: λ bc = λ*1*1 = λ.So λ = -2/9.Similarly for other equations, they would also give λ = -2/9. So it works. Thus, the critical point is at a = b = c =1.To verify if this is a maximum, we can check the second derivative or consider that perturbations around a = b = c =1 lead to a decrease in the function value, as seen in test cases. Therefore, the maximum of f is 1, achieved when a = b = c =1, hence for all positive a,b,c with abc=1, the sum is ≤1.But is this sufficient? To confirm that this is indeed the global maximum, we might need to check other possible critical points or use the fact that the function is symmetric and convex/concave.Alternatively, using the method of substitution and AM-GM.Let me consider the following approach:We need to show that:1/(1 + a + b) + 1/(1 + b + c) + 1/(1 + c + a) ≤1.Let’s apply the Cauchy-Schwarz inequality in the following form:For each denominator 1 + a + b, we can note that (1 + a + b)(1 + 1 + 1) ≥ (sqrt(1) + sqrt(a) + sqrt(b))^2.But not sure.Alternatively, use the following inequality: For positive real numbers x, y, z,1/(x + y + z) ≤ (1/(3))(1/x + 1/y + 1/z).But this is not generally true. For example, if x = y = z =1, then 1/3 ≤ 1/3, equality. But if x=1, y=1, z=2, then 1/4 ≤ (1 +1 +0.5)/3 ≈0.833.../3≈0.277, but 1/4 =0.25 ≤0.277, which holds. Hmm, but I'm not sure if this can be applied here.Alternatively, another idea: Use the substitution c =1/(ab). Let's denote S = 1/(1 + a + b) + 1/(1 + b + 1/(ab)) + 1/(1 + 1/(ab) + a).We need to show S ≤1.Let me make substitution a = x, b = y, so c =1/(xy). Then S becomes:1/(1 + x + y) + 1/(1 + y + 1/(xy)) + 1/(1 + 1/(xy) + x).This seems complicated, but maybe we can simplify terms.Let’s compute the second term: 1/(1 + y + 1/(xy)) = 1/(1 + y + 1/(xy)).Multiply numerator and denominator by xy:xy / (xy + xy*y +1 ) = xy / (xy + xy^2 +1 ).Similarly, third term: 1/(1 +1/(xy) +x) = 1/(1 + x +1/(xy)).Multiply numerator and denominator by xy:xy / (xy + x*xy +1 ) = xy / (xy + x^2 y +1 ).Therefore, S becomes:1/(1 +x + y) + xy/(xy + xy^2 +1 ) + xy/(xy +x^2 y +1 ).Not sure if this helps. Maybe try to find a common pattern or factor.Alternatively, consider that the denominators for the second and third terms are similar. Let me factor out xy:Denominator for second term: xy(1 + y) +1,Denominator for third term: xy(1 + x) +1.So S = 1/(1 +x + y) + xy/(xy(1 + y) +1 ) + xy/(xy(1 +x ) +1 ).Hmm, maybe substituting z = xy. Since c =1/(xy), then z =1/c.So z = xy, and since abc =1, c =1/(xy), so z =1/c.Then S becomes:1/(1 +x + y) + z/(z(1 + y) +1 ) + z/(z(1 +x ) +1 ).But with z = xy, which is a product. This might not help.Alternatively, maybe set t = x + y, s = xy. Then since z = s, but again, not sure.Alternatively, consider symmetry by setting x = y. Let’s assume x = y, so a = b =x, then c =1/x^2. Then S becomes:1/(1 +2x) + 1/(1 +x +1/x^2 ) +1/(1 +1/x^2 +x).Let’s compute this for x =1: 1/3 +1/(1 +1 +1) +1/(1 +1 +1 )=1/3 +1/3 +1/3=1.For x=2: 1/5 +1/(1 +2 +1/4) +1/(1 +1/4 +2)=1/5 +1/(3.25) +1/3.25≈0.2 +0.3077 +0.3077≈0.815.For x=1/2:1/(1 +1) +1/(1 +0.5 +4) +1/(1 +4 +0.5)=0.5 +1/5.5 +1/5.5≈0.5 +0.1818 +0.1818≈0.8636.In both cases, sum is less than 1. So when variables are equal, we get the maximum.This suggests that the maximum occurs at a =b =c =1, hence the inequality holds.But how to formalize this intuition?Perhaps using the method of smoothing or majorization. If we can show that any deviation from a =b =c =1 would decrease the sum, then the maximum is at 1.Alternatively, use the concept of convexity. If the function is convex or concave, then the extremum is at the symmetric point.But given the complexity of the function, this might be difficult.Wait, another approach: Let's use the substitution a = x^2, b = y^2, c = z^2, so that x^2 y^2 z^2 =1, i.e., xyz=1. Then the inequality becomes:1/(1 +x^2 + y^2) +1/(1 +y^2 + z^2) +1/(1 +z^2 +x^2) ≤1.Now, applying the Titu's lemma (Cauchy-Schwarz):Sum [1/(1 +x^2 + y^2)] ≤ [ (1 +1 +1)^2 ] / [ (1 +x^2 + y^2) + (1 +y^2 + z^2) + (1 +z^2 +x^2) ) ] = 9 / [ 3 + 2(x^2 + y^2 + z^2) ].So we have:Sum ≤ 9 / [3 + 2(x^2 + y^2 + z^2) ].We need to show that this is ≤1, which requires that 9 ≤ 3 + 2(x^2 + y^2 + z^2),i.e., 2(x^2 + y^2 + z^2) ≥6,i.e., x^2 + y^2 + z^2 ≥3.But since xyz=1, by AM-GM:x^2 + y^2 + z^2 ≥ 3*(x^2 y^2 z^2)^{1/3} =3*(1)^{2/3}=3.Therefore, equality holds when x=y=z=1. Therefore, Sum ≤9/[3 +2*3]=9/9=1.Therefore, the inequality holds.Wait, this seems promising. Let me verify.Yes, this approach works. Let me recap:Using Titu's lemma (Cauchy-Schwarz in the form of sum 1/a_i ≤ (sum 1)^2 / sum a_i), we have:Sum [1/(1 +x^2 + y^2)] ≤ 9 / [sum (1 +x^2 + y^2)].Sum in the denominator is 3 + 2(x^2 + y^2 + z^2).Since xyz=1, by AM-GM, x^2 + y^2 + z^2 ≥3*(x^2 y^2 z^2)^{1/3} =3.Therefore, denominator ≥3 +2*3=9. Therefore, the entire expression ≤9/9=1.Hence, the inequality is proven.But wait, this is using the substitution a =x^2, b =y^2, c =z^2. But in the original problem, a, b, c are positive real numbers with abc=1. So substituting a =x^2, b =y^2, c =z^2, we have x^2 y^2 z^2=1, so xyz=1 or xyz=-1. But since variables are positive, xyz=1. Therefore, this substitution is valid.Therefore, this approach works. Thus, the inequality is proven.Final Answerboxed{1}
✅:Given positive numbers (a, b, c) such that (a cdot b cdot c = 1), we need to prove the inequality:[frac{1}{1+a+b} + frac{1}{1+b+c} + frac{1}{1+c+a} leq 1] Key Steps and Reasoning:1. Substitution and Homogenization: - We use the substitution (a = x^2), (b = y^2), (c = z^2) such that (x^2 y^2 z^2 = 1), which simplifies to (xyz = 1) since (a, b, c) are positive. - The inequality transforms to: [ frac{1}{1 + x^2 + y^2} + frac{1}{1 + y^2 + z^2} + frac{1}{1 + z^2 + x^2} leq 1 ]2. Application of Titu's Lemma (Cauchy-Schwarz Inequality): - Applying Titu's lemma, we get: [ sum frac{1}{1 + x^2 + y^2} leq frac{(1 + 1 + 1)^2}{sum (1 + x^2 + y^2)} ] - Simplifying the denominator: [ sum (1 + x^2 + y^2) = 3 + 2(x^2 + y^2 + z^2) ]3. Using AM-GM Inequality: - By the AM-GM inequality, (x^2 + y^2 + z^2 geq 3 sqrt[3]{x^2 y^2 z^2} = 3) since (xyz = 1). - Therefore, the denominator becomes: [ 3 + 2(x^2 + y^2 + z^2) geq 3 + 2 cdot 3 = 9 ]4. Final Calculation: - Substituting back, we get: [ sum frac{1}{1 + x^2 + y^2} leq frac{9}{9} = 1 ]Thus, the inequality is proven, and the final answer is:[boxed{1}]
🔑:1. We begin by introducing a substitution that simplifies the problem. Let: [ a = x^3, quad b = y^3, quad c = z^3 ] Given that (abc = 1), substituting (a), (b), and (c) yields: [ x^3 cdot y^3 cdot z^3 = (xyz)^3 = 1 implies xyz = 1 ]2. Next, we need to show that: [ frac{1}{1 + a + b} + frac{1}{1 + b + c} + frac{1}{1 + c + a} leq 1 ] By substituting (a = x^3), (b = y^3), and (c = z^3) into the inequality: [ frac{1}{1 + x^3 + y^3} + frac{1}{1 + y^3 + z^3} + frac{1}{1 + z^3 + x^3} ] Now we need to bound each term (frac{1}{1 + x^3 + y^3} ).3. To proceed, we utilize the algebraic inequality for the cubes of (x) and (y). Specifically, we use the factorization and non-negativity of the expression (x^3 + y^3): [ x^3 + y^3 geq (x + y)xy ] This inequality is a consequence of the identity (x^3 + y^3 = (x + y)(x^2 - xy + y^2)) and the non-negativity of (x^2 - xy + y^2).4. We then rewrite the first term (frac{1}{1 + x^3 + y^3}) using this inequality: [ frac{1}{1 + x^3 + y^3} leq frac{1}{1 + (x + y)xy} ] Substituting (xyz = 1 implies xy = frac{1}{z}): [ frac{1}{1 + (x + y)xy} = frac{1}{1 + (x + y)frac{1}{z}} = frac{1}{frac{xz + yz + z}{z}} = frac{z}{x + y + z} ]5. By applying the result obtained for the first term, similarly simplifying the remaining two terms yields: [ frac{1}{1 + y^3 + z^3} leq frac{x}{y + z + x} ] [ frac{1}{1 + z^3 + x^3} leq frac{y}{z + x + y} ]6. Summing these three inequalities, we have: [ frac{1}{1 + x^3 + y^3} + frac{1}{1 + y^3 + z^3} + frac{1}{1 + z^3 + x^3} leq frac{z}{x + y + z} + frac{x}{y + z + x} + frac{y}{z + x + y} ] Since the denominators of the fractions on the right-hand side are all equal and sum to the whole, their sum is 1: [ frac{z}{x + y + z} + frac{x}{y + z + x} + frac{y}{z + x + y} = 1 ] Conclusion: [ boxed{frac{1}{1 + a + b} + frac{1}{1 + b + c} + frac{1}{1 + c + a} leq 1} ]
❓:Find the fraction (frac{p}{q}) with the smallest possible natural denominator for which (frac{1}{2014} < frac{p}{q} < frac{1}{2013}). Enter the denominator of this fraction in the provided field.
🧠:Okay, so I need to find the fraction p/q between 1/2014 and 1/2013, right? And the denominator q has to be the smallest possible natural number. Hmm, let's think about how to approach this.First, let me write down the inequality: 1/2014 < p/q < 1/2013. Both 2013 and 2014 are consecutive numbers, so their reciprocals are very close to each other. The fraction we're looking for must lie in that narrow interval. Since the denominators are large, the fraction p/q has to be a really small number, just a bit bigger than 1/2014 and a bit smaller than 1/2013.The question is asking for the fraction with the smallest possible denominator q. So, I need to find the minimal q such that there's an integer p where 1/2014 < p/q < 1/2013. To minimize q, maybe I can check denominators starting from 1 upwards until I find one that allows a numerator p to satisfy the inequality.Wait, but starting from q=1 might take a long time. Maybe there's a smarter way. I remember something about Farey sequences or mediant properties. Let me recall. The mediant of two fractions a/b and c/d is (a+c)/(b+d). It's a fraction that lies between them if they are neighbors in some Farey sequence. So maybe the mediant of 1/2014 and 1/2013 could be a candidate?Calculating the mediant: (1+1)/(2014+2013) = 2/4027. Let me check where this falls. 2/4027 compared to 1/2013.5 (since 2013.5 is the average of 2013 and 2014). Wait, 2/4027 is equal to 1/2013.5. But since 2013.5 is between 2013 and 2014, then 1/2013.5 would be between 1/2014 and 1/2013. So 2/4027 is indeed in the interval. But wait, 4027 is quite a large denominator. Since we're looking for the smallest possible q, maybe there's a fraction with a smaller denominator.Alternatively, maybe continued fractions can help here. The interval between 1/2014 and 1/2013 is very narrow. Let's think about how to approximate numbers in this interval with fractions of smaller denominators.Alternatively, since the two bounds are 1/(n+1) and 1/n where n=2013. So, maybe the fraction between them with the smallest denominator is related to continued fractions or the Farey sequence of order n. Wait, but Farey sequences of order n include all fractions with denominators up to n. But since n here is 2013, which is quite large, so the Farey sequence between 1/2014 and 1/2013 would have denominators up to 2014. But we need a denominator smaller than that.Alternatively, maybe consider the continued fraction expansion of the mid-point of the interval. Let me compute the mid-point first. The mid-point between 1/2014 and 1/2013 is (1/2013 + 1/2014)/2. Let me compute that:First, 1/2013 ≈ 0.000496756, 1/2014 ≈ 0.000496525. The average would be approximately 0.0004966405. But maybe exact fractions:(1/2013 + 1/2014)/2 = (2014 + 2013)/(2*2013*2014) = (4027)/(2*2013*2014). Hmm, not sure if that helps.Alternatively, maybe use the concept of continued fractions to find the simplest fraction between those two. The continued fraction for 1/2013 is [0; 2013], and for 1/2014 is [0; 2014]. But since they are adjacent integers, their continued fractions terminate immediately. The mediant was 2/4027, which is a continued fraction [0; 2013, 2], perhaps? Not sure.Alternatively, maybe the fraction with the smallest denominator between two fractions a and b can be found using the Farey sequence algorithm. The method involves taking mediants until you find one in the interval. But starting with 0 and 1, but here we have two very small fractions. Alternatively, since 1/2013 and 1/2014 are consecutive terms in the Farey sequence of order 2014, their mediant would be the next term in the Farey sequence of higher order, but maybe not the minimal denominator.Alternatively, perhaps consider solving the inequality 1/2014 < p/q < 1/2013 for integers p and q, with q minimal. Let's rearrange the inequality:From 1/2014 < p/q, we get q < 2014p.From p/q < 1/2013, we get p < q/2013.So combining these, we have:q/2014 < p < q/2013.So p must be the smallest integer greater than q/2014 and less than q/2013. For each q starting from 1 upwards, check if there's an integer p in that interval.So the approach is: for q=1, check if there's an integer p where 1/2014 < p/1 <1/2013. But 1/2013 ≈ 0.000496756, so p would have to be 0, but 0 is not greater than 1/2014. So no solution.q=2: check if there's p such that 2/2014 < p < 2/2013. 2/2014 ≈ 0.000993, 2/2013≈0.0009935. So p must be 1, but 1/2 = 0.5, which is way larger than 1/2013. So no.q=3: 3/2014 ≈ 0.00149, 3/2013≈0.00149. So p must be 1, since 0.00149 < p < 0.00149. Wait, but 3/2014 and 3/2013 are the bounds. Wait, 3/2014 ≈ 0.001489, and 3/2013 ≈ 0.001490. So the interval for p is (0.001489, 0.001490). So p must be 1, but 1/3 ≈ 0.3333, which is way larger. So no.q=4: 4/2014 ≈ 0.001986, 4/2013 ≈ 0.001986. Similarly, p must be 1, which gives 0.25, still way larger. So same issue.Wait, this seems like for small q, p/q is too big. Because 1/2013 is approximately 0.000496, so any p/q where p is 1 and q is less than 2013 would give a fraction larger than 1/2013, which is outside the upper bound. So maybe the first possible q where p/q can be less than 1/2013 is when q is greater than 2013. But 2013 is a huge denominator, but we need the minimal q. Wait, but the problem says "smallest possible natural denominator", so maybe q is somewhere in the middle.Wait, perhaps there's a better way. Since 1/2013 and 1/2014 are very close, the fraction between them with the minimal denominator is likely to be a continued fraction convergent or something similar. Let me recall that between any two numbers, the fraction with the minimal denominator can be found using continued fractions or the extended Euclidean algorithm.Alternatively, consider the difference between 1/2013 and 1/2014: 1/2013 - 1/2014 = (2014 - 2013)/(2013*2014) = 1/(2013*2014). So the interval is of width 1/(2013*2014). So any fraction p/q in that interval must satisfy |p/q - 1/2013| < 1/(2013*2014). By the theory of Diophantine approximation, such fractions p/q must satisfy q > 2013*2014 / (q + 2014). Wait, maybe not. Let me think again.Alternatively, using the concept that if two fractions a/b and c/d are consecutive in a Farey sequence, then bc - ad = 1. Here, 1/2013 and 1/2014 are neighbors in some Farey sequence. Let me check: 1/2013 and 1/2014. Their determinant is 1*2014 - 1*2013 = 1, so they are indeed adjacent in the Farey sequence of order 2014. So the next fraction inserted between them in the Farey sequence would be their mediant, which is (1+1)/(2013+2014)=2/4027, as I thought before. But 4027 is a larger denominator. But maybe there's a fraction with a smaller denominator that was already present in a lower-order Farey sequence.Wait, but if they are adjacent in the Farey sequence of order 2014, then any fraction between them must have a denominator greater than 2014. But that contradicts the possibility of having a smaller denominator. Hmm, maybe I need to think again.Wait, no. Farey sequences include all fractions between 0 and 1 with denominators up to n. So, in the Farey sequence of order n, fractions are between 0 and 1. So, 1/2013 and 1/2014 are in the Farey sequence of order 2014. Their mediant is 2/4027, which is in the Farey sequence of order 4027. However, there might be other fractions between them with smaller denominators that are not in the Farey sequence of order 2014 because their denominators are larger than 2014. Wait, no, if the denominator is smaller than 2014, then the fraction would have been included in the Farey sequence of order 2014. But since 1/2013 and 1/2014 are adjacent in that Farey sequence, there can't be any fraction between them with a denominator ≤2014. Therefore, any fraction between them must have a denominator >2014. But that seems contradictory because the problem is asking for a denominator smaller than that. Wait, but 2014 is already in the denominator of the lower bound. Hmm, perhaps there's a mistake here.Wait, let's clarify. If two fractions a/b and c/d are adjacent in the Farey sequence of order n, then any fraction between them must have a denominator greater than n. So in this case, since 1/2013 and 1/2014 are adjacent in the Farey sequence of order 2014, any fraction between them must have denominator greater than 2014. Therefore, the minimal possible denominator would be 4027, which is their mediant. But the problem states "smallest possible natural denominator", so maybe 4027 is the answer. But that seems too straightforward. However, the problem says "with the smallest possible natural denominator", which might not necessarily be in the Farey sequence. Wait, but according to Farey sequence properties, there cannot be any fraction between them with a smaller denominator. So if that's the case, then the answer would be 4027. But that seems counterintuitive because 4027 is quite large. Maybe there's a different approach.Wait, perhaps the Farey sequence theory applies to fractions in reduced form. So, 1/2013 and 1/2014 are in reduced form. Their mediant is 2/4027, which is also in reduced form since 2 and 4027 are coprime (4027 is a prime? Let me check: 4027 divided by 2 is 2013.5, not integer. Divided by 3: 3*1342=4026, so 4027 is 1342*3 +1, so not divisible by 3. 5: ends with 7, nope. 7: 7*575=4025, 4027-4025=2, not divisible by 7. 11: 11*366=4026, 4027-4026=1, not divisible by 11. So 4027 might be prime. If that's the case, then 2/4027 is reduced, and it's the only fraction between 1/2013 and 1/2014 with denominator 4027. But according to Farey sequences, there are no fractions between them with denominators less than 4027. So maybe the answer is 4027.But let's test with an example. Suppose we have 1/3 and 1/2. Their mediant is 2/5. But there is 1/2, 2/5, 1/3. Between 1/3 and 1/2, 2/5 is in between, but also 3/8, 3/7, etc. Wait, actually, Farey sequence of order 3 between 1/3 and 1/2 is nothing, but in higher orders, you get more. However, the minimal denominator between 1/3 and 1/2 is 2/5, which has denominator 5. But wait, there's also 3/7, which is between 1/3 (~0.333) and 1/2 (0.5). 3/7 ≈0.428, which is between them. But 3/7 has denominator 7, which is larger than 5. So 2/5 is the minimal denominator. So the mediant gives the minimal denominator between two adjacent Farey fractions.Therefore, following this logic, between 1/2013 and 1/2014, the minimal denominator fraction should be their mediant 2/4027. Therefore, the answer would be 4027. But let's verify this with another approach.Alternatively, suppose we need to find the minimal q such that there exists p where 1/2014 < p/q <1/2013. Let's rearrange the inequalities:From 1/2014 < p/q, we have q < 2014 p.From p/q <1/2013, we have p < q /2013.Therefore, combining these:q /2014 < p < q /2013.We need p to be an integer, so the interval (q/2014, q/2013) must contain at least one integer. Therefore, the length of the interval is q/2013 - q/2014 = q (1/2013 - 1/2014) = q/(2013*2014). For this interval to contain an integer, its length must be at least 1. Wait, but q/(2013*2014) >=1 => q >=2013*2014=2013*2014= Let's compute that: 2013*2014. Since 2013*2014 = 2013² +2013= (2000+13)² +2013= 2000² +2*2000*13 +13² +2013= 4,000,000 + 52,000 +169 +2013= 4,000,000 +52,000=4,052,000; 4,052,000 +169=4,052,169; 4,052,169 +2013=4,054,182. So q >=4,054,182. But this is way larger than 4027. Wait, that contradicts the earlier conclusion. So which is correct?Wait, no, the length of the interval (q/2014, q/2013) is q/(2013*2014). For this interval to contain at least one integer, the length must be at least 1. Therefore, q/(2013*2014) >=1 => q>=2013*2014=4,054,182. But this would mean that the minimal q is 4,054,182, which is way larger. But that can't be, since we already have 2/4027≈0.0004967, which is between 1/2014≈0.000496525 and 1/2013≈0.000496756. So there's a contradiction here.Therefore, my previous reasoning must be flawed. The interval length being less than 1 doesn't necessarily mean there's no integer in it. For example, if the interval is between two non-integers but spans an integer. For example, if the interval is (0.9,1.1), length 0.2 <1, but contains the integer 1. So, the correct condition is not that the length must be >=1, but rather that the ceiling of the lower bound is less than the floor of the upper bound. That is, ceil(q/2014) < floor(q/2013). Which would imply that there's an integer p between them.So, to find the minimal q such that ceil(q/2014) < floor(q/2013). Let's formalize this.We need an integer p such that q/2014 < p < q/2013.Which can be rewritten as:There exists integer p where floor(q/2013 - 1) >= p >= ceil(q/2014 +1).Wait, perhaps not. Let me think again. For a given q, the values of p must satisfy q/2014 < p < q/2013. Therefore, the smallest possible p is floor(q/2014) +1, and the largest possible p is ceil(q/2013 -1). For there to exist such a p, we need floor(q/2014) +1 <= ceil(q/2013 -1). Hmm, this is getting complicated.Alternatively, for a given q, compute the lower bound q/2014 and the upper bound q/2013. Then, if there's an integer between them, then that q is acceptable. So, the minimal q is the smallest q where the interval (q/2014, q/2013) contains an integer.To find such q, we can use the concept of Beatty sequences or look for q such that the fractional part of q/2014 is less than the fractional part of q/2013, but I'm not sure.Alternatively, set up the inequality:There exists integer p where q/2014 < p < q/2013.Multiply all parts by 2013*2014 to eliminate denominators:q*2013 < p*2013*2014 < q*2014.So,q*2013 < p*2013*2014 < q*2014.Divide all parts by 2013:q < p*2014 < q*2014/2013.Hmm, not sure if helpful.Alternatively, rearrange the inequality q/2014 < p < q/2013 as:q/2014 < p and p < q/2013.Which implies:q/2014 < p < q/2013.Therefore,q/2014 < q/2013 - ε, where ε is a small positive number.But this might not be helpful.Alternatively, think of it as:Find q such that there's an integer p where:(1) p > q/2014(2) p < q/2013Let me define k = floor(q/2013). Then, p must be at least ceil(q/2014 +1). Wait, maybe not.Alternatively, define the interval for p as (q/2014, q/2013). The length of this interval is q/(2013*2014). So, as q increases, the length increases. We need the interval to contain at least one integer. The minimal q for which this happens is the minimal q such that the interval contains an integer.This is similar to the problem of finding the minimal q such that the interval (q*a, q*b) contains an integer, where a=1/2014 and b=1/2013. The minimal such q is the minimal denominator of a fraction between a and b.This seems related to the concept of the ceiling function. Let's think in terms of continued fractions. Maybe the minimal solution can be found using the continued fraction of the interval's midpoint.Alternatively, another approach: we can model this as a Diophantine inequality. We need to find integers p, q such that:2013p < q < 2014pSo, given that, for some integer p, q must be between 2013p and 2014p.But since q must be an integer, and 2013p and 2014p are consecutive multiples, the only integer q in (2013p, 2014p) is q = 2013p +1, 2013p +2, ..., 2014p -1.But then, substituting back, we have p/q must satisfy 1/2014 < p/q <1/2013.So for each p, we can look for q such that 2013p < q <2014p.But since p and q are positive integers, for each p, q must be in that interval. The minimal q would then be 2013p +1 for the smallest possible p such that 2013p +1 <2014p, which simplifies to 1 < p. Therefore, p must be at least 2. Then, q would be 2013*2 +1=4027. Which gives p=2, q=4027, which is the mediant fraction. So this suggests that the minimal q is 4027.Wait, but if p=1, then q must be between 2013*1=2013 and 2014*1=2014. So q=2014, but p=1, q=2014 gives 1/2014, which is the lower bound, not included. Similarly, q=2013 gives 1/2013, the upper bound, not included. So p=1 cannot be used. For p=2, q must be between 4026 and 4028. So q=4027 is the only integer, which gives 2/4027≈0.0004967, which is between 1/2014≈0.000496525 and 1/2013≈0.000496756. Therefore, this is valid. Therefore, q=4027 is indeed the minimal denominator. Hence, the answer is 4027.But wait, earlier when I considered q=4027, the mediant, I thought it might be the answer. But in the first approach, I was confused because Farey sequences suggested that any fraction between them must have a denominator greater than 2014, and 4027 is greater than 2014. But in this second approach, by solving the inequalities, we indeed find that the minimal q is 4027. So that must be the answer.Therefore, after going through this extensive reasoning, I conclude that the denominator is 4027.
✅:To find the fraction (frac{p}{q}) with the smallest natural denominator (q) such that (frac{1}{2014} < frac{p}{q} < frac{1}{2013}), we analyze the interval and use properties of Farey sequences and mediants.1. Mediant Property: The mediant of (frac{1}{2014}) and (frac{1}{2013}) is (frac{2}{4027}). This fraction lies strictly between the two given fractions because: [ frac{1}{2014} approx 0.000496525 < frac{2}{4027} approx 0.0004967 < frac{1}{2013} approx 0.000496756 ]2. Minimal Denominator Verification: For smaller denominators (q), the interval (left(frac{q}{2014}, frac{q}{2013}right)) must contain an integer (p). However, checking denominators (q = 1, 2, ldots, 4026) shows no valid (p) exists within the interval. The first valid (p) occurs at (q = 4027), where (p = 2) satisfies the inequality.3. Conclusion: The fraction (frac{2}{4027}) is the simplest fraction between (frac{1}{2014}) and (frac{1}{2013}) with the smallest denominator.Answer: (boxed{4027})
🔑:To find the fraction ( frac{p}{q} ) with the smallest possible natural denominator ( q ) such that [frac{1}{2014} < frac{p}{q} < frac{1}{2013},]we can follow these steps:1. Understanding the Inequality: [ frac{1}{2014} < frac{p}{q} < frac{1}{2013}. ] This implies: [ frac{1}{2013} - frac{1}{2014} > 0. ] 2. Finding a Common Denominator: Let's convert the right-hand side of the inequality to have the same denominator: [ frac{1}{2013} = frac{2014}{2013 times 2014} ] and [ frac{1}{2014} = frac{2013}{2013 times 2014}. ] Therefore, we rewrite: [ frac{2013}{2013 times 2014} < frac{p}{q} < frac{2014}{2013 times 2014}. ]3. Determine the Fraction: We need to find the fraction ( frac{p}{q} ) that lies between these two values with the smallest possible ( q ). Notice that: [ frac{2013}{2013 times 2014} < frac{1}{2013} < frac{2014}{2013 times 2014}. ] This means the smallest integer ( q ) would be the denominator in a simple fraction larger than (2014): Let’s calculate: [ frac{2013+1}{2014} = frac{2014}{2014} = 1 ] Since we are looking for a value between the two given fractions:4. Using Farey Sequence Property: By knowing the fractions: - ( frac{2013}{2013 times 2014} = frac{2013}{4032062} ) - ( frac{2014}{2013 times 2014} = frac{2014}{4032062} ) Continuing the fraction closer, we find: [ frac{p}{q} = frac{1}{2013} ] The only possible natural comparison with the smallest numerator would be ( q = 2013 ), the next integer number forming the fraction (frac {2014}{4039} ) From the given translation, using the Farey sequence the denominator involves two closest natural numbers in sequence forming the consecutive forms. Hence, [ q = 4027 ] Conclusion:[boxed{4027}]